ML14080A507

From kanterella
Jump to navigation Jump to search
Final Written Examination with Answer Key (401-5 Format) (Folder 3)
ML14080A507
Person / Time
Site: Indian Point Entergy icon.png
Issue date: 02/19/2014
From: David Silk
Operations Branch I
To:
Entergy Nuclear Indian Point 2
Jackson D
Shared Package
ML13252A147 List:
References
UO1890
Download: ML14080A507 (207)


Text

Exam Quest Numb Answer Exam Quest Numb Answer 1A 51 D 2C 52 8 3C 53 8 48 54 8 50 55 c 68 56 c 7C 57 c 8A 58 D 98 59 8 10 A. 60 A 11 8 61 8 12 D 62 c 13 D 63 c 14 c - 64 c

.~~

~16A 65 D 66 A 17 c 67 c 18 D 68 8 19 B. 69 c

~ 20 C. 70 c

~ 22 A 71 72 D

c 23 8 73 8 24 8 74 c 25 c 75 D 26 c 76 A 27 D 77 c 28 A 78 c 29 A 79 8 30 D 80 8 31 D 81 D 32 8 82 A 33 8 83 A 34 8 84 c 35 A 85 c 36 D 86 8 37 c 87 D.

38 D 88 A 39 c 89 8 40 D 90 c 41 D 91 D 42 c 92 D 43 A 93 8 44 c 94 8 45 D 95 8 46 D 96 D 47 8 97 8 48 A 98 D 49 c 99 D 50 8 100 A

Exam Outline Cross

Reference:

Level RO SRO Tier# 1 Group# 1 KIA# 000007K103 Knowledge of the operational implications of the following concepts as they apply to the reactor trip: -Reasons for closing the main turbine governor valve and the main turbine stop valve after a reactor trip Importance 3. 7 4 Question# 1 Which of the following is the basis for verifying the Main Turbine Tripped after a reactor trip?

A. Prevent consequences resulting from an excessive cooldown of the RCS.

B. Prevent an unnecessary transition to FR-H.1 upon exit from E-0.

C. Prevent missile hazards resulting from turbine overspeed condition.

D. Prevent exceeding RCS- SG ~p limit.

Answer: A Explanation/Justification:

A. Correct B. Incorrect. Plausible because continuing to remove steam from the steam generators will deplete inventory. This can satisfy SG level entry conditions to FR-H.1

C. Incorrect. Plausible because turbine overspeed is a possible consequence if a generator trip occurs before a turbine trip.

D. Incorrect. Plausible because if all inventory is removed from a SG the pressure will rapid decrease. This condition can challenge the RCS-SG delta P limit.

Technical

References:

2-E-0 BG Proposed References to be provided: None Learning Objective 12LP-ILO-EOPEOO 1 Question Source: New Question History: NA Question Cognitive Level: Fundamental Knowledge 10 CRF Part 55 Content: 55.41 (b) 5 Comments

Exam Outline Cross

Reference:

Level RO SRO Tier# 1 Group# 1 KIA# 000008A108 Ability to operate and/or monitor the following as they apply to the Pressurizer Vapor Space Accident: - PRT level pressure and temperature Importance 3.8 3.8 Question# 2 The unit is at 100% power and PZR Pressure has lowered to 2025 psig in two minutes. Which of the following sets of conditions could confirm that a PORV is leaking?

1. Acoustic Monitor Indication for flow through the PORV
2. Tailpipe temperature at saturation temperature for PRT pressure
3. Tailpipe temperature at saturation temperature for the PZR
4. PRT level elevated
5. PRT temperature slightly elevated
6. PRT temperature equal to tailpipe temperature A. 1,3,6 B. 3,4,6 C. 2,4,5 D. 1,2,4,5 Answer: C Exp Ia nation/Justification:

For this event, items 2,4, and 5 will occur and the others will not occur. No more than 2 wrong answers can be eliminated by knowing that any one of the incorrect changes is incorrect. All choices appear twice.

Acoustic monitor indication is only available for the PZR safeties, but it is plausible that an operator makes this mistake especially since the panel is in the back of the control room.

Tailpipe temperature being at saturation for the PZR is wrong but is plausible if misread or if the candidate misunderstands where temperature is measured.

PRT temperature equal to tailpipe temperature is wrong but plausible if the candidate does not know that this is liquid temperature.

Technical

References:

Syst Desc 1 .4 Proposed References to be provided: None Learning Objective 12LP-ILO-RCSPZR 4 Question Source: New Question History: NA Question Cognitive Level: Comprehension 10 CRF Part 55 Content: 55.41 (b) 5 Comments

Exam Outline Cross

Reference:

Level RO SRO Tier# 1 Group# 1 KIA# 000011 K1 01 Knowledge of the operational implications of the following concepts as they apply to the Large Break LOCA: - Natural circulation and cooling, including reflux boiling Importance 4.1 4.4 Question# 3 Which of the following describes the reflux cooling mechanism during a large break LOCA and what conditions are necessary to enhance this method of cooling?

A. Steam produced in the core rises in the hot legs to the steam generators where it is condensed and returned to the vessel via the cold legs. Steam generator levels must be maintained to enhance reflux cooling.

B. Steam produced in the core rises in the hot legs to the steam generators where it is condensed and returned to the vessel via the cold legs. RCS pressure must be maintained less than SG pressure to ensure reflux cooling will occur.

C. Steam produced in the core flows along the top of the hot leg piping to the steam generators where it is condensed and returns to the vessel via the bottom of the hot leg piping. Steam generator levels must be maintained to enhance reflux cooling.

D. Steam produced in the in the core flows along the top of the hot leg piping to the steam generators where it is condensed and returns to the vessel via the bottom of the hot leg piping. RCS pressure must be maintained less than SG pressure to ensure reflux cooling will occur.

Answer: C

Explanation/Justification:

A. Incorrect. Plausible because steam produced in the core rises to the SG and is condensed; however water flows back down the hot leg to the core. Also it is true that SG level must be maintained.

B. Incorrect. Plausible because steam produced in the core rises to the SG and is condensed; however water flows back down the hot leg to the core. Also if RCS is less than SG pressure heat will be transferred from the SG to the RCS.

C. Correct.

D. Incorrect. Plausible because steam produced in the core rises to the SG and is condensed, and the water flows back to the core via the hot leg; however, if RCS is less than SG pressure heat will be transferred from the SG to the RCS.

Technical

References:

2-ECA-1.1 BG Proposed References to be provided: None Learning Objective 12LP-ILO-NACIRC 1 12LP-ILO-NACIRC 3 Question Source: Bank Question History: NA Question Cognitive Level: Comprehension 10 CRF Part 55 Content: 55.41 (b) 5 Comments

Exam Outline Cross

Reference:

Level RO SRO Tier# 1 Group# 1 KIA# 000015A112 Ability to operate and/or monitor the following as they apply to the Reactor Coolant Pump Malfunctions: - Reactor coolant loop flow meters Importance 2.8 3.1 Question# 4 Given:

  • The unit is operating at 25% power during a plant shutdown for refueling.
  • 23 RCP shaft sheers.

Which of the following correctly describes the plant status and indicated RCS flow for loop 23?

Reactor Status Indicated flow A. Tripped ==0%

B. Tripped ==30%

C. Not Tripped ==0%

D. Not Tripped ==30%

Answer: 8

Explanation/Justification:

The plant response will be that low flow will stop in 23 loop and then there will be backflow indicating about 30%. The plant will trip due to 2/3 loop flow instruments on% loops being low.

There are multiple reasons why it is plausible that an operator may not realize the plant is going to trip. First there will be no RCP breaker input into the Rx Trip circuitry. Another possible point of confusion is that power is just above the P-8 setpoint where the coincidence for low flow trip goes from % to 2/4.

The flow instruments are loop elbow taps that respond the same to flow in either direction. It is plausible that a candidate could believe that reverse flow will not show on the meter. It's additionally possible that a candidate could mistakenly believe there will be no flow with the shaft sheared.

Technical

References:

Proposed References to be provided: None Learning Objective 12LP-ILO-RCS001 13 12LP-ILO-RCS001 4 Question Source: New Question History: NA Question Cognitive Level: Comprehension 10 CRF Part 55 Content: 55.41 (b) 3 Comments

Exam Outline Cross

Reference:

Level RO SRO Tier# 1 Group# 1 KJA# 000022K304 Knowledge of the reasons for the following responses as they apply to the Loss of Reactor Coolant Makeup: - Isolating letdown Importance 3.2 3.4 Question# 5 The plant is 100% normal full power lineup when the running charging pump trips. The cause is a common pump suction issue. It will be 5 minutes before a charging pump can be started.

What will be done with normal letdown in accordance with AOP-CVCS-1 due to these conditions and why?

A. The CVCS Beds will be bypassed to ensure temperature limits are not exceeded.

B. No action should be necessary since the Non Regen Heat Exchanger will maintain letdown temperature for a short duration loss of charging.

C. Letdown will be diverted to ensure that VCT level and temperature limits are not exceeded.

D. Letdown will be isolated to prevent flashing downstream of the letdown orifices.

Answer: D

Explanation/Justification:

Per 2-AOP-CVCS-1 steps 4.7/4.8, letdown is isolated whenever there is no charging pump running at this point in the procedure. It is plausible that for a short duration loss of charging with expected restoration, the procedure could allow continued letdown flow. However, the procedure clearly does not allow this.

A. Plausible since bed overheating is a legitimate concern, but procedure does not allow this.

B. Plausible since for 5 minutes is a short term loss of charging and CCW will be cooling letdown at the NRHX, but procedure does not allow this.

C. Plausible since VCT parameters are a concern.

D. Correct answer.

Technical

References:

2-AOP-CVCS-1 Proposed References to be provided: None Learning Objective 12LP-ILO-AOPCVC 2 Question Source: New Question History: NA Question Cognitive Level: Fundamental Knowledge 10 CRF Part 55 Content: 55.41 (b)4 Comments

Exam Outline Cross

Reference:

Level RO SRO Tier# 1 Group# 1 KJA# 000025K205 Knowledge of the interrelations between the Loss of Residual Heat Removal System and the following:- Reactor building sump Importance 2.6 2.6 Question# 6 Given:

  • Vacuum fill is in progress in accordance with 2-SOP-1.1.1, Vacuum Filling and Degassing the Reactor Coolant System.
  • The RCS is at 62'3" elevation
  • 21 RHR pump is in service
  • 22 RHR pump is in standby.

The operating RHR pump tripped on overcurrent and the standby pump failed to start. Assuming no change in RHR pump status, how will the core be cooled 24 hours2.777778e-4 days <br />0.00667 hours <br />3.968254e-5 weeks <br />9.132e-6 months <br /> from now?

A. Two charging pumps taking a suction on the VCT to the RCS via normal cold leg injection.

B. One Recirc pump taking a suction on the Recirc Sump to RHR Heat exchanger and back to the RCS.

C. One Recirc Pump taking a suction on the Recirc Sump to RHR Heat exchanger to Sl Pump suction and back to RCS via hot leg injection.

D. One Sl pump taking a suction on the RWST to the RCS via normal cold leg injection.

Answer: 8

Explanation/Justification:

With the RCS intact (Vacuum fill in progress) in accordance with AOP-RHR-1, an Sl pump is used to pump the contents of the RWST through the RCS & PRZR to the PRT. When the PRT rupture disk ruptures the RWST water will flow to the Recirculation Sump. It will take approximately 15 hours1.736111e-4 days <br />0.00417 hours <br />2.480159e-5 weeks <br />5.7075e-6 months <br /> to pump the contents of the RWST into containment.

A. Incorrect. Plausible because charging pumps can be aligned to makeup to the RCS in AOP-RHR-1, Loss of RHR; however, the charging pumps are NOT aligned to the VCT.

B. Correct. This lineup is correct if the condition lasts for greater approximately 15 hours1.736111e-4 days <br />0.00417 hours <br />2.480159e-5 weeks <br />5.7075e-6 months <br />. (i.e., after the RWST was pumped through the RCS to the sump).

C. Incorrect. Plausible because this lineup would be correct if the condition lasted for greater than approximately 15 hours1.736111e-4 days <br />0.00417 hours <br />2.480159e-5 weeks <br />5.7075e-6 months <br /> and an RCS cold leg was NOT intact.

D. Incorrect. Plausible because with the RCS intact (Vacuum fill in progress), an Sl pump is used to pump the contents of the RWST through the RCS & PRZR to the PRT. When the PRT rupture disk ruptures the RWST water will flow to the Recirculation Sump.

Technical

References:

2-AOP-RHR-1 Proposed References to be provided: None Learning Objective 12LP-ILO-AOPRHR 2 Question Source: New Question History: NA Question Cognitive Level: Comprehension 10 CRF Part 55 Content: 55.41(b)10 Comments

Exam Outline Cross

Reference:

Level RO SRO Tier# 1 Group# 1 KIA# 000026K302 Knowledge of the reasons for the following responses as they apply to the Loss of Component Cooling Water: - The automatic actions (alignments) within the CCWS resulting from the actuation of the ESFAS Importance 3.6 3.9 Question# 7 The plant is in a normal full power lineup. During I&C troubleshooting, a technician inadvertently causes a Train A Safety Injection actuation. What effect will this have on the Component Cooling Water System?

AuxCCW CCWfrom CCWPumps Pumps RHRHx 2 Pumps 1 Pump Both Valves A. Running Running Open Both Pumps Both Valves B. All Running Running Open 1 Pump One Valve C. All Running Running Open 2 Pumps Both Pumps One Valve D. Running Running Open Answer: C

Explanation/Justification:

A safety injection signal from either train will start all three component cooling water pumps.

A safety injection signal from one train will only start the train specific Aux CCW pump and open the train specific CCW outlet valve from the RHR heat exchanger. Because either train of safety injection will start all 3 CCW pumps and Aux CCW pumps and CCW from RHR Heat Exchanger Valve are started/opened from a safety injection signal, any combination of pumps and valves is plausible.

A. Incorrect.

B. Incorrect.

C. Correct.

D. Incorrect Technical

References:

Plant Prints Proposed References to be provided: None Learning Objective 12LP-ILO-ESS001 5 Question Source: Modified Question History: NA Question Cognitive Level: Fundamental Knowledge 10 CRF Part 55 Content: 55.41 (b) 7 Comments

Exam Outline Cross

Reference:

Level RO SRO Tier# 1 Group# 1 KIA# 0000572136 Conduct of Operations-Knowledge of procedures and limitations involved in core alterations.

Importance 3 4.1 Question# 8 Given the following plant conditions on Unit 2:

  • The unit is in Mode 6 with refueling activities in progress.
  • Containment purge is in service.
  • MCC 24/24A is out of service for breaker PM
  • 22 Static Inverter faults.

Which of the following describes a consequences of the loss of 22 Static Inverter?

A. Suspend core alterations until 32 source range channel is re-energized from alternate source.

B. Refueling operations can continue for 1 hour1.157407e-5 days <br />2.777778e-4 hours <br />1.653439e-6 weeks <br />3.805e-7 months <br /> if RCS boron concentration is verified greater than limit in COLR.

C. Refueling operations may continue as long as R-41 is restored to OPERABLE status in 7 days.

D. Suspend core alterations until Audio Count Rate/Scaler Time is re-energized.

Answer: A

Exp Ia nation/Justification:

A. Correct.

B. Incorrect. Plausible because refueling can continue for one hour under certain conditions; however loss of one SR NIS is not one of them.

C. Incorrect. Plausible because R-41 monitors containment particulate activity; however, R-41 is energized from 24/24A instrument buses. The 7 day completion time is plausible because grab samples could be used.

D. Incorrect. Plausible because Audio Count Rate/Scaler Timer is powered from an instrument bus; however it is powered from 18 24.

Technical

References:

2-AOP-IB-1 Proposed References to be provided: None Learning Objective 12LP-ILO-AOPIB1 5 Question Source: New Question History: NA Question Cognitive Level: Comprehension 10 CRF Part 55 Content: 55.41 (b) 6 Comments

Exam Outline Cross

Reference:

Level RO SRO Tier# 1 Group# 1 KIA# 000038A215 Ability to determine and interpret the following as they apply to a SGTR: - Pressure at which to maintain RCS during S/G cool down Importance 4.2 4.4 Question# 9 The plant is preparing to cool down following a SGTR. Given the following plant conditions:

  • Safety Injection has been terminated
  • Pressurizer Level is 68% and increasing
  • Ruptured SG NR Level is 35% and decreasing
  • No other event is in progress

A. Increase RCS Makeup Flow B. Turn ON Pressurizer Heaters C. Increase AFW flow to ruptured SG D. Lower ruptured SG Pressure Answer: B

Explanation/Justification:

A. Incorrect. Plausible because increasing makeup would slowly raise RCS pressure stopping the backfill and this would be used if Pressurizer level was low.

B. Correct.

C. Incorrect. Plausible because this action is used if SG level approximately 14%

during SG cooldown activities.

D. Incorrect. Plausible because this action lowering SG pressure would result in increase SG level and decrease PRZR level; however, this is not directed in E-3.

Technical

References:

2-ES-3.2 3-ES-3.2 Proposed References to be provided: None Learning Objective 13LP-ILO-EOPE30 16 Question Source: Bank Question History: Unit 3 NRC 2013 Question Cognitive Level: Comprehension 10 CRF Part 55 Content: 55.41 (b) 10 Comments

Exam Outline Cross

Reference:

Level RO SRO Tier# 1 Group# 1 KIA# 000040K101 Knowledge of the operational implications of the following concepts as they apply to Steam Line Rupture: - Consequences of PTS Importance 4.1 4.4 Question # 10 15 minutes ago, the plant experienced a main steam line break, from 100%

power. Because of difficulties in closing the MSIVs, two SGs have blown dry.

Current Plant status is as follows:

  • Step 6 of E-2, "Faulted SG Isolation", is in progress.
  • RCS temperature is 285°F and decreasing.
  • Sl flow is still being supplied to the RCS,
  • Total AFW flow is 800 gpm.
  • All RCPs have been stopped due to loss of cooling water.
  • RCS Pressure is 1000 psig and stable
  • Attachment 1 of E-0 has been completed Which of the following is of greatest immediate concern?

A. A crack could propagate in the reactor vessel.

B. Natural circulation cooling will be limited.

C. A significant reduction in the heat sink capacity is present.

D. The loss of thermal driving head in the dry SGs will reduce the amount of natural circulation flow.

Answer: A

Explanation/Justification:

A. Correct. Crack propagation is the possible result of a rapid cooldown combined with elevated pressure.

B. Incorrect. Plausible because at 1000 psig, accumulator injection is close and loss of natural circulation is possible; however, with pressure stable it is not the greatest immediate concern.

C. Incorrect. Plausible because loss of 2 steam generators for cooldown will increase the time to achieve cold shutdown, but only one SG is necessary to safely remove all decay heat.

D. Excessive cooldown in one or more loops will impede natural circulation; however, natural eire will be re-established if the RCS heats up.

Technical

References:

2-FR-P.1 BG Proposed References to be provided: None Learning Objective 12LP-ILO-EOPFP1 3 Question Source: Bank Question History: NA Question Cognitive Level: Comprehension 10 CRF Part 55 Content: 55.41(b)10 Comments

Exam Outline Cross

Reference:

Level RO SRO Tier# 1 Group# 1 KIA# 000058K301 Knowledge of the reasons for the following responses as they apply to the Loss of DC Power: - Use of de control power by ED/Gs Importance 3.4 3.7 Question # 11 An electrical short causes a loss of 23 DC Power Panel. Subsequently a loss of the Station Auxiliary Transformer occurs. Five minutes after these events, what would the configuration of the Emergency Diesel Generators be?

A. All three EDGs would be running with the output breaker for 3A open.

B. All three EDGs would be running with their output breakers closed.

C. Only 21 and 23 EDGs would be running with their output breakers open.

D. Only 22 and 23 EDGs would be running with their output breakers closed.

Answer: B Explanation/Justification:

A. Incorrect. Plausible because all 3 EDGs will be running. 23 DC Power Panel supplies Normal DC control power to 22 EDG, but 22 EDG has emergency control power from Distribution Panel 22. Similarly, Power Panel 23 supplies normal control power to 480V bus 3A breakers, and they are backed up from Power Panel21.

B. Correct. Control Power for the diesel generators and their output breakers have normal and backup DC control power supply via an Auto Bus Transfer switch.

C. Incorrect. Plausible because 22 EDG receives Normal control power from 23 DC Power Panel; backup power from 22 Distribution Panel will start the EDG.

Similarly, the output breakers have a normal and auto backup control power supply.

D. Incorrect. Plausible because 21 EDG receives backup control power from 23 DC Power Panel; it will start on its normal control power supply. Similarly, the output breakers have a normal and auto backup control power supply.

Technical

References:

2-AOP-DC-1 Proposed References to be provided: None Learning Objective 12LP-ILO-EDS03 11 Question Source: Bank Question History: NA Question Cognitive Level: Analysis Synthesis Evaluation 10 CRF Part 55 Content: 55.41 (b) 7 Comments

Exam Outline Cross

Reference:

Level RO SRO Tier# 1 Group# 1 KIA# 0000552417 Emergency Procedures/Plan -

Knowledge of EOP terms and definitions.

Importance 3.9 4.3 Question # 12 Given the following conditions:

  • A Station Blackout has occurred.
  • The Team is performing ECA-0.0, Loss of All AC Power.
  • RCS Subcooling is 0 degrees F.
  • PZR Pressure 1800 psig
  • A RED Path exists on the Heat Sink CSF Status Tree.
  • 21 EDG has been started and is supplying its associated bus.
  • The Team is preparing to transition to the appropriate recovery procedure.

Which ONE (1) of the following describes the recovery strategy?

A. Transition to FR-H.1 upon exit from ECA-0.0. Perform ECA-0.1, Loss of All AC Power Recovery Without Sl Required, when FR-H.1 is complete.

B. Transition to FR-H.1 upon exit from ECA-0.0. Perform ECA-0.2, Loss of All AC Power Recovery with Sl Required, when FR-H.1 is complete.

C. Transition to ECA-0.1, Loss of All AC Power Recovery Without Sl Required, and enter FR-H.1 when directed by ECA-0.1.

D. Transition to ECA-0.2, Loss of All AC Power Recovery with Sl Required, and enter FR-H.1 when directed by ECA-0.2.

Answer: D

Explanation/Justification:

This question satisfies the KA because it requires knowledge of what the term "RED path on Heat Sink CSF Status tree" means. The question also requires knowledge of the procedures for ECA-0.0 and ECA-0.2 and when Red Path conditions are addressed.

A. Incorrect. Plausible because FR-H.1 is the correct procedure for a RED Path condition on Heat Sink; however, the procedure is not entered until after step 9 of ECA-0.1 or step 11 of ECA-0.2 is complete.

B. Incorrect. Plausible because FR-H.1 is the correct procedure for a RED Path condition on Heat Sink; however, the procedure is not entered until after step 9 of ECA-0.1 or step 11 of ECA-0.2 is complete.

C. Incorrect. Plausible because ECA-0.1 would be the correct procedure if no safety injection signal or condition existed. Candidate should recognize that 0 degrees subcooling requires a safety injection.

D. Correct.

Technical

References:

2-ECA-0.0 2-ECA-0.0 BG 2-FR-H.1 2-FR-H.1 BG 3-ECA-0.0 3-FR-H.1 Proposed References to be provided: None Learning Objective 12LP-ILO-EOPROU 10 Question Source: Bank Question History: NA Question Cognitive Level: Comprehension 10 CRF Part 55 Content: 55.41 (b) 10 Comments

Exam Outline Cross

Reference:

Level RO SRO Tier# 1 Group# 1 KIA# 000056A133 Ability to operate and/or monitor the following as they apply to the Loss of Offsite Power: - PORV block valve control switch Importance 3.3 3.5 Question # 13 Unit 2 was operating at 100% when the following occurred:

  • A loss of 480V Bus 6A occurred due to a fault.
  • The bus could not be re-energized.
  • Following completion of all actions of AOP-480V-1, an area wide blackout occurred.
  • All equipment responded as designed for the blackout.
  • Five minutes after the blackout, no additional operator actions have been taken at this point.

If it is desired to use a PZR PORV to control pressure, what actions will be required?

A. Either PORV can be used. The PORV Block Valves will need to be opened using the flight panel switches.

B. One PORV is available. No operation of the associated PORV Block Valve switch is necessary since the Block Valve switch was maintained in the OPEN position per AOP-480V-1.

C. No PORV is available. Following reset of MCCs, the associated PORV Block Valve can opened using the flight panel switch.

D. One PORV will be available after its associated PORV Block Valve is opened. The associated PORV Block can be opened at this time using the flight panel switch.

Answer: D

Explanation/Justification:

IP-2 operates with block valves closed and the switches in automatic. The power supplies for these valves are Sa and 6A. A loss of 6A will cause one block to fail closed. On the loss of offsite power, the other valve will lose power, but only during the time before the emergency diesel re-energizes bus SA.

A. Incorrect but plausible. It is plausible that a candidate might forget that action is normally needed to use a PORV. Also a candidate could believe that AOP-480V-1 took action to get block valves open.

B. Incorrect but plausible. It is plausible that AOP-480V-1 would open the block valve that has power. Saying the switch is maintained in the open position makes this choice incorrect. This remains plausible since other switches (e.g. PZR BU Heaters) have a locked in open/on position.

C. Incorrect but plausible. Plausible because a candidate may not know which MCCs power the block valves or they may not know that these do not strip.

D. Correct answer.

Technical

References:

Syst Desc 1.4 Proposed References to be provided: None Learning Objective 12LP-ILO-RCSPZR 2 Question Source: New Question History: NA Question Cognitive Level: Comprehension 10 CRF Part 55 Content: 55.41 (b) 7 Comments

Exam Outline Cross

Reference:

Level RO SRO Tier# 1 Group# 1 KIA# 0000542416 Emergency Procedures/Plan -

Knowledge of EOP implementation hierarchy and coordination with other support procedures or guidelines such as, operating procedures, abnormal operating procedures, severe accident management guidelines.

Importance 3.5 4.4 Question # 14 Given:

  • The reactor has tripped from 100% power
  • Level is at 52% WR in all SGs and lowering slowly
  • Pressurizer level is 14% and lowering slowly
  • AC 480V Bus 6A Normal Supply Breaker 52/6A opened on a bus overcurrent
  • 21 AFW pump is out of service for maintenance
  • 22 AFW pump just tripped on overspeed What action is or will be required?

A. Actuate Sl and Go To E-0, "Reactor Trip or Safety Injection.

B. Remain in ES-0.1, "Reactor Trip Response" perform AOP-FW-1, "Loss of Main Feedwater", in parallel.

C. Go to FR-H.1, "Response to Loss of Secondary Heat Sink".

D. Remain in ES-0.1, "Reactor Trip Response" perform AOP-480V-1, "Loss of Normal Power to any 480V Bus", in parallel.

Answer: C

Explanation/Justification:

A. Incorrect. Plausible because Pressurizer level is significantly lower than expected, but it is not below the Sl initiation criterion.

B. Incorrect. Plausible WR level is well above the feed and bleed setpoint and AOP-FW-1 will initiate actions to restore AFW flow; however, FR-H.1 conditions are met and it is a higher priority procedure.

C. Correct. 52% wide range SG level is approximately 0% narrow range level.

The criteria to enter FR-H. 1 are met.

D. Incorrect. Similar to B above AOP-480V-1 will potentially re-energize bus 6A which will provide power to 23 AFW pump.

Technical

References:

2-FR-H.1 2-FR-H.1 BG 3-FR-H.1 Proposed References to be provided: None Learning Objective 12LP-ILO-EOPFH1 3 Question Source: Modified Question History: NA Question Cognitive Level: Comprehension 10 CRF Part 55 Content: 55.41 (b) 10 Comments

Exam Outline Cross

Reference:

Level RO SRO Tier# 1 Group# 1 KIA# 000065A207 Ability to determine and interpret the following as they apply to the Loss of Instrument Air:- Whether backup nitrogen supply is controlling valve position Importance 2.8 3.2 Question # 15 Consider the following plant conditions:

  • Loss of all AC power has occurred and is affecting the entire site.
  • The Turbine Driven ABF pump is providing flow to the SGs.
  • A rapid cooldown has been commenced.

During the cooldown, what are the expected methods of control for the following:

(1) 21-24 SG Turbine Driven ABFP flow control valves (FCV-405A, B, C&D) and (2) 21-24 Atmospheric Steam Dumps (PCV-1134 through 1137)

A. (1) Manual remote (CCR) control using nitrogen.

(2) Manual local control using nitrogen B. (1) Manual local control using nitrogen (2) Manual local control using nitrogen C. (1) Manual remote (CCR) control using nitrogen (2) Manual remote (CCR) control using nitrogen.

D. (1) Manual local control using nitrogen (2) Manual remote (CCR) control using nitrogen.

Explanation/Justification:

A. Incorrect: In a complete loss of AC power instrument and station air would be lost. Critical components have either safety related air or nitrogen. For AFW, nitrogen automatically aligns to supply and the CCR retains control. For the Atmospheric Steam Dumps nitrogen is aligned locally and the valves are controlled from the control room.

B. Incorrect: Plausible: because methods both are potential (NOT Expected) ways to take control of the valves.

C. Correct:

D. Incorrect: Plausible because AFW valves can be controlled locally but it is not the Expected method. Atmospheric Steam dumps are expected to be controlled from the CCR.

Technical

References:

2-AOP-AIR-1 2-ECA-0.0 Syst Desc 18 Syst Desc 21 Proposed References to be provided: None Learning Objective 12LP-ILO-MFW001 15 12LP-ILO-MFW001 9 12LP-ILO-MSS001 15 12LP-ILO-MSS001 8 Question Source: Bank Question History: Unit 2 NRC 2008 Question Cognitive Level: Comprehension 10 CRF Part 55 Content: 55.41 (b) 7 Comments

Exam Outline Cross

Reference:

Level RO SRO Tier# 1 Group# 1 KIA# OOWE04K202 Knowledge of the interrelations between the LOCA Outside Containment and the following: -

Facility's heat removal systems, including primary coolant, emergency coolant, the decay heat removal systems, and relations between the proper operation of these systems to the operation of the facility Importance 3.8 4 Question# 16 Given the following:

  • A LOCA outside containment has occurred.
  • The crew is performing the actions in ECA-1.2, LOCA Outside Containment.

Which ONE (1) of the following actions will be attempted to isolate the break and which indication is used to determine if the leak has been isolated in accordance with ECA-1.2?

A. Isolate RHR Cold Leg Injection piping; RCS pressure is monitored, because Sl flow will repressurize the RCS with the break isolated.

B. Isolate RHR Cold Leg Injection piping; PZR level is monitored, because with the break isolated, RCS inventory will rapidly rise.

C. Isolate Sl Hot Leg Injection piping; RCS pressure is monitored, because Sl flow will repressurize the RCS with the break isolated.

D. Isolate Sl Hot Leg Injection piping; PZR level is monitored, because with the break isolated, RCS inventory will rapidly rise.

Answer: A

Exp Ian ation/Justification:

KA match:

The candidate has to understand that some of the actions taken in ECA-1.2 will actually decrease decay heat removal capability (i.e. isolation of cold leg injection piping during a LOCA).

Plausibility:

ECA-1.2 will close the cold leg injection valves, but not hot leg injection valves.

RCS pressure is monitored to check for successful leak isolation. PZR level is not. Therefore A is the correct answer. It is plausible that the hot leg injection piping would be checked as a leak point. It is also plausible that the cold leg piping would not be isolated since it is in use for injection. Therefore both of these choices are plausible. It is also plausible that PZR level would used to check for success instead of pressure since it is likely that leak isolation would completely stop the LOCA and level would rapidly return in the PZR.

Therefore all choices are plausible, but only A is correct per ECA-1.2.

Technical

References:

2-ECA-1.2 Proposed References to be provided: None Learning Objective 12LP-ILO-EOPC12 4 Question Source: Bank Question History: IP2 2010 Question Cognitive Level: Comprehension 10 CRF Part 55 Content: 55.41 (b) 10 Comments

Exam Outline Cross

Reference:

Level RO SRO Tier# 1 Group# 1 KJA# OOWE05K201 Knowledge of the interrelations between the Loss of Secondary Heat Sink and the following: -

Components, and functions of control and safety systems, including instrumentation, signals, interlocks, failure modes, and automatic and manual features Importance 3. 7 3.9 Question # 17 Following a small-break LOCA and Sl actuation, core exit TC's read 625°F and increasing.

  • . RCS pressure is 1400 psia and rising.
  • S/G pressures are stable at 900 psig.
  • Containment pressure is stable at 3 psig.
  • T ave is offscale LOW
  • The control room operators are attempting to establish MFW flow in response to a loss of secondary heat sink.
  • They are unable to lift the live lead on the feed water isolation relay signal.

Under the above conditions, which of the following is TRUE?

A. Air control line for Main Feed Regulating Valves can locally bypass trip solenoids to open valves from the control room.

B. The Main Feed Regulating Valves can be locally operated using the manual handwheel.

C. Air control line for Low Flow Bypass Feed Regulating Valves can locally bypass trip solenoids to open valves from the control room.

D. The Main Boiler Feed Pumps cannot be reset.

Answer: C Explanation/Justification:

A. Incorrect. Plausible because this is true if the bypass control air line is NOT installed. If the line is installed, the bypass FRVs can be controlled from the control room.

B. Incorrect. The Main and Low Flow Bypass Valves fail closed on a loss of air.

The manual handwheels cannot override this. Local control requires air to drive the valve open. The handwheel adjust in the close direction.

C. Correct. The control air bypass line connects the positioner output directly to the valve diaphragm bypassing the trip solenoids. The valves can then be controlled from the control room.

D. Incorrect. Plausible because the Feed Pump must be reset to restart; however, it can be reset.

Technical

References:

2-FR-H.1 Proposed References to be provided: None Learning Objective 12LP-ILO-EOPFH1 5 Question Source: Modified Question History: Unit 2 NRC 2010 Question Cognitive Level: Comprehension 10 CRF Part 55 Content: 55.41 (b) 10 Comments

Exam Outline Cross

Reference:

Level RO SRO Tier# 1 Group# 1 KIA# OOWE11A201 Ability to determine and interpret the following as they apply to the Loss of Emergency Coolant Recirculation: -Facility conditions and selection of appropriate procedures during abnormal and emergency operations Importance 3.4 4.2 Question # 18 Which of the following are major action(s) of ECA-1.1, "LOSS OF EMERGENCY COOLANT RECIRCULATION"?

1. Provide guidance on aligning the Safety Injection Pump suction directly to the Containment Sump.
2. Terminate Cold Leg Recirculation and restore Charging.
3. Cooldown and depressurize the Reactor Coolant System to allow Residual Heat Removal to be put into service.
4. Provide methods to make-up to the Refueling Water Storage Tank.

A. 1, 2, and 3.

B. 4 ONLY.

C. 1, 3, and 4.

D. 3 and 4 ONLY.

Answer: D

Explanation/Justification:

Choice 1 is not done by ECA-1.1 but is plausible because this would be a way of cooling the core with a loss of RWST.

Choice 2 is not done by ECA-1.1 but is plausible because it would conserve RWST level.

Choice 3 is a correct action in ECA-1.1.

Choice 4 is a correct action in ECA-1.1.

Technical

References:

2-ECA-1.1 3-ECA-1.1 3-ECA-1.1 Background Proposed References to be provided: None Learning Objective 13LP-ILO-EOPE1 0 22 Question Source: Bank Question History: Unit 3 NRC 2010 Question Cognitive Level: Fundamental Knowledge 10 CRF Part 55 Content: 55.41 (b) 10 Comments

Exam Outline Cross

Reference:

Level RO SRO Tier# 1 Group# 2 KIA# OOWE10K302 Knowledge of the reasons for the following responses as they apply to the Natural Circulation with Steam Void in Vessel with/without RVLIS: - Normal, abnormal and emergency operating procedures associated with Natural Circulation with Steam Void in Vessel with/without RVLIS Importance 3.2 3.7 Question # 19 The Team is performing ES-0.4, (Natural Circulation Cooldown With Steam Void in Vessel (Without RVLIS)). Step 6 "Equalize Charging and Letdown" has been performed.

What is the purpose of equalizing Charging and Letdown?

A. So a void formation in the vessel will be minimized B. So changes in pressurizer level will be an indication of void formation C. So the pressurizer will not go water solid D. So letdown isolation I heater trip will not occur during depressurization Answer: B

Explanation/Justification:

A. Incorrect. Plausible because the procedure is entered with the expectation that a void exists. Charging and letdown are matched when plant conditions are stabilized to monitor void formation and growth.

B. Correct. At step 6 the cooldown is stopped. With subcooling established, any changes in PRZR level will be the result of the void formation not temperature change or inventory balance.

C. Incorrect. Plausible because void formations can cause the PRZR to become water solid; however at this time in the procedure, actions would have been taken to establish water level in the pressurizer.

D. Incorrect. Plausible because maintaining level above the heaters is desirable to maintain pressure control; however charging and letdown are matched when plant conditions are stabilized to monitor void formation and growth.

Technical

References:

2-ES-0.4 2-ES-0.4 BG 3-ES-0.4 Proposed References to be provided: None Learning Objective 12LP-ILO-EOPS04 1 Question Source: Bank Question History: NA Question Cognitive Level: Fundamental Knowledge 10 CRF Part 55 Content: 55.41 (b) 5 Comments

Exam Outline Cross

Reference:

Level RO SRO Tier# 1 Group# 2 KIA# 000003K110 Knowledge of the operational implications of the following concepts as they apply to Dropped Control Rod:-

Definitions of core quadrant power tilt Importance 2.6 2.9 Question # 20 Given:

  • The unit is in MODE 1 at 45% RTP during power ascension.
  • QPTR is determined to be 1.05.
  • Due to grid instability, the System Operator has requested the maximum power possible and when the unit can resume full power operations.

In addition to realigning the dropped rod, which one of the following describes the conditions that must be met to raise reactor power to 100% RTP?

A. QPTR must be lowered to at least 1.02 prior to raising reactor power above 50% RTP.

B. Reactor power may be raised to 100% RTP after determining that hot channel peaking factors are within limits.

C. Continue the power ascension but do not exceed 85% RTP until QPTR is lowered to at least 1.02.

D. Commence a power reduction until 30% RTP, then the power reduction must be terminated until a safety analysis is performed.

Answer: C

Explanation/Justification:

This satisfies the KA because it is looking for the definition of a core quadrant power tilt (i.e., > 1.03 quadrant power tilt ratio).

A. Incorrect. Plausible because the applicability for QPTR is > 50% power; however, the Required Actions allow operation for an unlimited period of time, which is permissible by LCO 3.0.4. Once the MODE of Applicability is entered, the Required Actions must be performed within the associated Completion Times.

B. Incorrect. Plausible because calculating Hot Channel Factors is required for a Quadrant Power Tilt condition; however, power cannot exceed 85% with a tilt of 1.05.

C. Correct. A quadrant power tilt condition exists with QPTR > 1.02. The Required Actions allow operation for an unlimited period of time, which is permissible by LCO 3.0.4. Once the MODE of Applicability is entered, the Required Actions must be performed within the associated Completion Times.

D. Incorrect. Plausible because TS action is to reduce power 3% Rated Thermal Power for every 1% QPTR exceeds 1.00. This would result in a 15% power reduction from 100% not 45%.

Technical

References:

Tech Specs Proposed References to be provided: None Learning Objective 12LP-ILO-ICNXC 11 Question Source: Bank Question History: NA Question Cognitive Level: Comprehension 10 CRF Part 55 Content: 55.41 (b) 6 Comments

Exam Outline Cross

Reference:

Level RO SRO Tier# 1 Group# 2 KIA# 000005K303 Knowledge of the reasons for the following responses as they apply to the Inoperable/Stuck Control Rod:- Tech-Spec limits for rod mismatch Importance 3.6 4.1 Question# 21 Initial Conditions:

  • A 20% load rejection from 100% has occurred and the crew is stabilizing the plant in accordance with the appropriate AOP.

Current Conditions:

  • Control Bank "D" Group Counters are at 180 steps.
  • H-2, a Control Bank "D" rod, indicates 223 steps on IRPI.
  • All other Control Bank "D" rods indicate 180 steps on IRPI.
  • I&C reports no blown fuse indicators for H-2.

Which ONE (1) of the following describes the current condition of rod H-2 and the basis for the applicable Tech Spec action?

Condition Basis A. Untrippable Shutdown Margin B. Untrippable Peaking Factors C. Trippable Shutdown Margin D. Trippable Peaking Factors Answer: A

Explanation/Justification:

A. Correct. The rod should have "ratcheted" into the core with a blown moveable gripper fuse and be at some level lower than the rest of the bank. The rod is untrippable.

B. Incorrect. Plausible because the rod is untrippable but the completion time and basis are incorrect. The completion time of 6 hours6.944444e-5 days <br />0.00167 hours <br />9.920635e-6 weeks <br />2.283e-6 months <br /> is the value to be in Mode 3 for this condition if SDM is not verified.

C. Incorrect. Plausible because the completion time to realign a misaligned rod is correct; however the rod is untrippable and thus inoperable by TS. The basis is also incorrect but plausible because misaligned rods can have an impact on AFD.

D. Incorrect. Plausible because the completion time of 6 hours6.944444e-5 days <br />0.00167 hours <br />9.920635e-6 weeks <br />2.283e-6 months <br /> is the value to be in Mode 3 for an untrippable rod if SDM is not verified.

Technical

References:

2-AOP-ROD-1 3-AOP-ROD-1 Tech Specs Proposed References to be provided: None Learning Objective 12LP-ILO-ICROD 14 Question Source: New Question History: NA Question Cognitive Level: Comprehension 10 CRF Part 55 Content: 55.41 (b) 5 Comments

Exam Outline Cross

Reference:

Level RO SRO Tier# 1 Group# 2 KIA# 000033A201 Ability to determine and interpret the following as they apply to the Loss of Intermediate Range Nuclear Instrumentation: -

Equivalency between source-range, intermediate-range, and power-range channel readings Importance 3 3.5 Question # 22 Unit 2 is conducting a reactor startup following a refueling outage. The following conditions exist:

  • Source Range Instrument N 31 indicates 2.1xE4 cps.
  • Source Range Instrument N 32 indicates 2.0xE4 cps.
  • Intermediate Range Instrument N 35 indicates 1.8xE-11 amps.
  • Intermediate Range Instrument N 36 indicates 1.0xE-10 amps.
  • Rods are in manual with no rod motion.
  • Source Range and Intermediate Range Nuclear Instruments are slowly rising.

Which ONE of the following best explains the indications?

A. N 35 compensating voltage is set too high B. N 35 compensating voltage is set too low C. N 36 compensating voltage is set too high D. N 36 compensating voltage is set too low Answer: A

Explanation/Justification:

A. Correct.

B Incorrect N 35 reads too low; C Incorrect Overlap is proper for N 36; D Incorrect Overlap is proper for N 36.

Technical

References:

GRAPH-NI-1 Proposed References to be provided: None Learning Objective 12LP-ILO-ICEXC 8 Question Source: Bank Question History: DC Cook NRC 2004 Question Cognitive Level: Comprehension 10 CRF Part 55 Content: 55.41 (b) 6 Comments

Exam Outline Cross

Reference:

Level RO SRO Tier# 1 Group# 2 KIA# 000036K101 Knowledge of the operational implications of the following concepts as they apply to Fuel Handling Incidents: - Radiation exposure hazards Importance 3.5 4.1 Question # 23 Given the following conditions:

  • The plant is in Refueling mode
  • Refueling is in progress
  • The Refueling SRO reports damage to an irradiated fuel assembly on the Refueling Crane mast
  • R-41 and R-42, Containment Radiation monitors, are in alarm
  • VC Purge is in progress Which ONE ( 1) of the following will be the status of the VC equipment and personnel in the VC immediately following implementation of the applicable AOP?

A. VC Purge Supply Valves are closed and Exhaust Valves remain open to maintain negative pressure in VC. All personnel are evacuated from the VC.

B. VC Purge Supply and Exhaust Valves are closed. All personnel are evacuated from the VC.

C. VC Purge Supply Valves are closed and Exhaust Valves remain open to maintain negative pressure in VC. Non-essential personnel are evacuated from the VC.

D. VC Purge Supply and Exhaust Valves are closed. Non-essential personnel are evacuated from the VC.

Answer: B

Explanation/Justification:

This is a modified bank question (bank id 24035). The question was modified to be more plausible since the original had choices where pressure relief valves remain open.

The correct answer per 2-AOP-FH-1 is B. Having the exhaust valves remain open to keep negative pressure in the VC is plausible since this is what is done in the FSB. This is more plausible than in the past because the VC hatch is removed during fuel movement (a contingency is in place to close VC). In the past the VC had modified integrity during fuel movement. It is also plausible that essential personnel would remain in the VC. Therefore, there is one correct answer and the other three choices are all plausible.

Technical

References:

2-AOP-FH-1 Proposed References to be provided: None Learning Objective 12LP-ILO-FHD001 11 Question Source: Modified Question History: NA Question Cognitive Level: Comprehension 10 CRF Part 55 Content: 55.41 (b) 11 Comments

Exam Outline Cross

Reference:

Level RO SRO Tier# 1 Group# 2 KIA# 000060K202 Knowledge of the interrelations between the Accidental Gaseous Radwaste Release and the following: -Auxiliary building ventilation system Importance 2. 7 3.1 Question # 24 Given the following conditions:

  • The plant is at 100% power
  • 23 Large Gas Decay Tank is aligned for in-service and re-use
  • 24 Large Gas Decay Tank is in standby
  • 22 Large Gas Decay Tank is isolated with a pressure of 90 psig and a content of 5000 Curies
  • All remaining Gas Decay Tanks are inerted with nitrogen Which one of the following describes the plant response if 22 Large Gas Decay Tank relief valve (1622) were to fail open?

A. Warn radiation level alarm on R-44, Plant Vent Air Monitor; Closes RCV-014 and Secures PAB Supply Fan, diverts PAB Exhaust through charcoal beds.

B. High radiation level alarm on R-44, Plant Vent Air Monitor ;

Secures VC Purge/Pressure Relief if in progress, diverts PAB Exhaust through charcoal beds C. Warn radiation level alarm on R-50, Waste Gas Decay Tank Monitor, Closes RCV-014 and Secures PAB Supply Fan, diverts PAB Exhaust through charcoal beds D. High radiation level alarm on R-50, Waste Gas Decay Tank Monitor, Secures PAB Supply Fan, secures VC Purge/Pressure Relief if in progress.

Answer: B

Exp Ia nation/Justification:

A. Incorrect. Plausible because the closing RCV-014 would be correct if it was opened and at the Alarm level and securing the supply fan would limit spread of potential contamination; however PAB Supply fan is not secured and RCV-014 would not be opened at this time.

B. Correct.

C. Incorrect. Plausible because securing the supply fan would limit spread of potential contamination. Diverting PAB exhaust through the charcoal beds is correct for an R-44 Alarm level condition. There are no automatic actions associated with R-50.

D. Incorrect. Plausible because securing the supply fan would limit spread of potential contamination. Securing VC Purge/Pressure Relief is correct for an R-44 Alarm level condition. There are no automatic actions associated with R-50.

Technical

References:

2-SOP-12.3.3 Proposed References to be provided: None Learning Objective 12LP-ILO-GWR01 5 12LP-ILO-RMS001 3 Question Source: Bank Question History: NA Question Cognitive Level: Comprehension 10 CRF Part 55 Content: 55.41 (b) 7 Comments

Exam Outline Cross

Reference:

Level RO SRO Tier# 1 Group# 2 KIA# 000069A202 Ability to determine and interpret the following as they apply to the Loss of Containment Integrity: -

Verification of automatic and manual means of restoring integrity Importance 3.9 4.4 Question# 25 Given the following conditions:

  • The plant is operating at 100% power.
  • Work performed on SI-AOV-1813 requires a Post Maintenance Retest.
  • SI-878A, 21 Spray Pump Discharge Test Isolation, must be opened for the retest. It is a locked closed, non-automatic containment isolation valve Which ONE (1) of the following is required to open SI-878A to perform the retest?

A. CRS or SM approval ONLY, since the valve is part of a line open to containment during normal operations B. An approved Temporary Change is required for the off-normal valve position.

C. A Dedicated Operator must be stationed at the valve to close it in the event of an emergency D. It must be inside of a tagged boundary to be opened above Mode 5.

Answer: C

Explanation/Justification:

A. Incorrect. Plausible because the CRS or SM must approve the action; however it is not the only action required.

B. Incorrect. Plausible because a Temporary Change could address the condition; however it is not required as long as actions are covered by another procedure or restest step text.

C. Correct.

D. Incorrect. Plausible because a tagged boundary would provide some form of protection; however it would not necessarily protect penetration open to containment.

Technical

References:

Tech Specs Proposed References to be provided: None Learning Objective 12LP-ILO-VCCIS 11 Question Source: Bank Question History: NA Question Cognitive Level: Comprehension 10 CRF Part 55 Content: 55.41 (b) 7 Comments

Exam Outline Cross

Reference:

Level RO SRO Tier# 1 Group# 2 KIA# 000074A112 Ability to operate and/or monitor the following as they apply to an Inadequate Core Cooling: - RCS temperature and pressure indicators Importance 4.1 4.4 Question # 26 Given the following plant conditions:

  • A loss of coolant accident has taken place.
  • The operators are transitioning to 2-ES-1.2, Post LOCA Cooldown and Depressurization, from 2-E-1, Loss of Reactor or Secondary Coolant.

The following plant conditions exist:

  • All Core Exit thermocouples are reading approximately 720°F and increasing.
  • RCS subcooling is 0°F
  • RVLIS NC Range indicates 35%.
  • All SG pressures are approximately 1125 psig
  • Containment pressure peaked at 3.5 psig
  • Total AFW flow is approximately 100 GPM
  • SG WR levels are 22%, 24%, 24%, and 31%, respectively Which ONE of the following procedures will the crew implement to respond to these conditions?

A. 2-FR-H.1, Response to Loss of Secondary Heat Sink B. 2-FR-H.2, Response to Steam Generator Overpressure C. 2-FR-C.1, Response to Inadequate Core Cooling D. 2-FR-C.2, Response To Degraded Core Cooling Answer: C

Explanation/Justification:

A. Incorrect. Plausible because the Heat Sink safety function is severely challenged (Red), but Core Cooling is a higher priority.

B. Incorrect. Plausible because SG pressure is above required value for entry to 2-FR-H.2, but Core Cooling is a higher priority C. Correct. Red condition for 2-FR-C.1 exists due to <30°F subcooling, 0 RCPs, Core Exit T/Cs >700 °F, and RVLIS <41%

D. Incorrect. Plausible because having either the RVLIS value or the GET value would require entry into FR-C.2. Having both conditions makes this choice incorrect.

Technical

References:

2-F-0.2 2-F-0.3 Proposed References to be provided: None Learning Objective 12LP-ILO-EOPROU 10 12LP-ILO-EOPROU 11 Question Source: Bank Question History: NA Question Cognitive Level: Comprehension 10 CRF Part 55 Content: 55.41 (b) 10 Comments

Exam Outline Cross

Reference:

Level RO SRO Tier# 1 Group# 2 KIA# OOWE03K202 Knowledge of the interrelations between the LOCA Cooldown and Depressurization and the following: -Facility's heat removal systems, including primary coolant, emergency coolant, the decay heat removal systems, and relations between the proper operation of these systems to the operation of the facility Importance 3. 7 4 Question # 27 A LOCA has occurred, and the control room operators are now preparing to conduct a post LOCA cooldown and depressurization. The plant is currently stable with the following conditions:

  • RCS pressure .... 1200 Psig
  • RCS Tavg ....... .450°F
  • PZR level. ...... visible in the indicating range
  • Total Sl flow ... 580 gpm It is decided that the plant will be cooled down at 100°F/hr by dumping steam to the condenser.

The cooldown will cause A. Sl flow to Decrease and RCS pressure to Decrease.

B. Sl flow to Increase but have no effect on RCS pressure.

C. Sl flow to Decrease and Pressurizer Level to Decrease.

D. Sl flow to Increase and RCS pressure to Decrease.

Answer: D

Explanation/Justification:

A. Incorrect. Plausible because RCS Pressure will decrease; however, Sl flow will increase B. Incorrect. Plausible because Sl flow will increase; however RCS pressure will decrease.

C. Incorrect. Plausible because Pressurizer level will decrease; however Sl flow will increase.

D. Correct. Sl flow will increase due to the reduction in pressure of the RCS.

Technical

References:

2-E-1 BG Proposed References to be provided: None Learning Objective 12LP-ILO-SIS01 4 Question Source: Bank Question History: NA Question Cognitive Level: Comprehension 10 CRF Part 55 Content: 55.41 (b) 5 Comments

Exam Outline Cross

Reference:

Level RO SRO Tier# 2 Group# 1 KIA# 003000K103 Knowledge of the physical connections and/or cause-effect relationships between the RCPS and the following systems: - RCP seal system Importance 3.3 3.6 Question # 28 The following plant conditions exist:

Reactor Power: 100%

21 RCP Seal Return Flow: 0.2 gpm and stable 21 RCP Upper Radial Bearing Temperature: 145°F and stable 21 RCP Vibration: 12 mils and increasing @ 2 mil/hr 21 RCP #1 Seal Inlet Temperature: 165 F and increasing 21 RCP Stator Winding Temperature: 175°F and stable The following annunciators are in alarm:

  • 21 RCP HIGH VIBRATION
  • RCP NO. 1 SEAL RETURN HIGH/LOW FLOW (COMMON)

The CRS has announced entry into AOP-RCP-1.

What actions should be taken based upon 21 RCP indications?

A. Trip the reactor, Trip 21 RCP, Initiate E-0, Close 21 RCP Seal Return Valve when pump has stopped rotating.

B. Initiate Attachment 1,Initiate POP 3.1 (Plant Shutdown From Full Power Operation to Zero Power Condition), Trip 21 RCP within 8 hours9.259259e-5 days <br />0.00222 hours <br />1.322751e-5 weeks <br />3.044e-6 months <br />.

C. Notify I&C to change High Vibration alarm setpoint, Initiate Attachment 1, Initiate POP 3.1 (Plant Shutdown From Full Power Operation to Zero Power Condition).

D. Trip the reactor, Trip 21 RCP, Go to E-0.

Answer: A Explanation/Justification:

A. Correct.

B. Incorrect. Plausible because depending on the severity of the conditions, a plant shutdown may be initiated as opposed to a reactor trip.

C. Incorrect. Plausible because the action is correct if the rate of increase is less than 1 mil/hr.

D. Incorrect. Plausible because this action is correct (GO TO E-0) if the reason for the trip is a failure of #2 or 3 seal combined with high vibrations.

Technical

References:

2-AOP-RCP-1 Proposed References to be provided: None Learning Objective 12LP-ILO-RCSRCP 15 Question Source: Bank Question History: NA Question Cognitive Level: Comprehension 10 CRF Part 55 Content: 55.41 (b) 7 Comments

Exam Outline Cross

Reference:

Level RO SRO Tier# 2 Group# 1 KIA# 004000K631 Knowledge of the effect of a loss or malfunction on the following CVCS components: - Seal injection system and limits on flow range Importance 3.1 3.5 Question # 29 Given:

  • The unit is at 100% power.
  • Pressurizer Level Channel 2 is in control.
  • The diaphragm on LT-460 develops a large leak.

Which of the following describes the effect of this failure?

A. Pressurizer level indicates high.

Seal Injections flow decreases.

B. Pressurizer Level indicates high Seal Injection flow increases C. Pressurizer level indicates low Seal Injection flow decreases D. Pressurizer level indicates low Seal Injection flow increases Answer: A

Explanation/Justification:

A. Correct B. Incorrect. Plausible because LT-460 is the controlling channel and failure will cause indicated level to fail high. Failure high will cause charging pump speed to decrease resulting in decreased seal injection flow.

C. Incorrect. Plausible because candidate must determine that the diaphragm failure will result in an increase in indicate pressurizer level. It is correct that seal injection flow will decrease.

D. Incorrect. Plausible because candidate must determine that the diaphragm failure will result in an increase in indicate pressurizer level. A decrease controlling pressurizer level will result in an increase in charging speed and seal injection flow.

Technical

References:

Syst Desc 3 Proposed References to be provided: None Learning Objective 12LP-ILO-AOPINT 5 Question Source: New Question History: None Question Cognitive Level: Comprehension 10 CRF Part 55 Content: 55.41 (b)7 Comments

Exam Outline Cross

Reference:

Level RO SRO Tier# 2 Group# 1 KJA# 005000K112 Knowledge of the physical connections and/or cause-effect relationships between the RHRS and the following systems: -

Safeguard pumps Importance 3.1 3.4 Question # 30 A large break LOCA has occurred 10 hours1.157407e-4 days <br />0.00278 hours <br />1.653439e-5 weeks <br />3.805e-6 months <br /> ago, and your crew has just returned from days off to relieve the watch. A plant cooldown is in progress. As you begin your walkdown of the CCR, what would you expect the safety injection system lineup to be at this point in time?

A. Recirculation pump in service- through the RHR heat exchangers- to the cold leg injection valves.

B. Recirculation pump in service -through the RHR heat exchangers -to the hot leg injection valves.

C. Recirculation pump in service- through the RHR heat exchangers- to the safety injection pumps - to the cold leg injection valves.

D. Recirculation pump in service - through the RHR heat exchangers -to the safety injection pumps- to both the hot and cold leg injection valves.

Answer: D

Exp Ia nation/Justification:

A Incorrect. Plausible because this lineup is correct if transfer to hot leg Recirc has not been completed yet.

B. Incorrect. Plausible because Recirc pumps through heat exchangers is correct; however, the flowpath to hot leg Recirc is through the Sl pumps.

C. Incorrect. Plausible because the flowpath from Recirc pumps through Sl pumps is correct; however the cold leg injection valves are not correct.

D. Correct.

Technical

References:

2-E-1 Proposed References to be provided: None Learning Objective 12LP-ILO-EOPS13 4 13LP-ILO-EOPE10 22 Question Source: Bank Question History: NA Question Cognitive Level: Comprehension 10 CRF Part 55 Content: 55.41 (b) 7 Comments

Exam Outline Cross

Reference:

Level RO SRO Tier# 2 Group# 1 KIA# 006000A107 Ability to predict and/or monitor changes in parameters (to prevent exceeding design limits) associated with operating the ECCS controls including: -

Pressure, high and low Importance 3.3 3.6 Question # 31 When implementing ECA-1.1, LOSS OF EMERGENCY COOLANT RECIRCULATION, you are instructed to depressurize the Steam Generators (SG) to 710 psig. What is the basis for stopping the SG depressurization at 710 psig?

This pressure corresponds to the Reactor Coolant System pressure A. at the point where the Safety Injection Accumulators are required to be isolated.

B. below which the high head injection flow paths must be isolated.

C. at the point where low head injection sources begin to inject.

D. below which the Safety Injection Accumulators will begin to inject.

Answer: D

Explanation/Justification:

A. Incorrect. Plausible because 710 is above the pressure at which the accumulator would start to inject and typically the accumulators are isolated at this pressure. In ECA-1.1 the accumulators are slowly injected to "buy time" to restore a low head injection path.

B. Incorrect. Plausible because the cooldown in ECA-1.1 is rapid which may result in a PTS concern. Isolating high pressure injection would minimize the potential for repressurization.

C. Incorrect. Plausible because it is true that low pressure injection would begin at a pressure slightly below this pressure; however, low pressure injection is not the concern; the concern is low pressure recirculation.

D. Correct. The goal of the SG depressurization is to set the plant up for slow controlled accumulator injection to "buy time" until recirculation flow path can be established.

Technical

References:

2-ECA-1.1 BG Proposed References to be provided: None Learning Objective 12LP-ILO-EOPC11 1 Question Source: Bank Question History: NA Question Cognitive Level: Fundamental Knowledge 10 CRF Part 55 Content: 55.41 (b) 10 Comments

Exam Outline Cross

Reference:

Level RO SRO Tier# 2 Group# 1 KIA# 007000A301 Ability to monitor automatic operation of the PRTS, including:

-Components which discharge to the PRT Importance 2. 7 2.9 Question # 32 The Unit is operating at 100% power with all control systems in Automatic. The following conditions have been noted from the initial time of the event, and after 1 hour:

Initially 1 Hour Later PRT level 70% 75%

PRT Pressure 5 psig 6 psig Pressurizer Tailpipe Temperature 150°F 150°F Acoustic Monitors NOT Alarming NOT Alarming Given the above information, which one of the events below best explains the noted change in plant conditions?

A. Leakage by a Pressurizer PORV or Safety Valve has increased.

B. PCV-135, Letdown Backpressure Control Valve, is controlling letdown backpressure too HIGH.

C. RCP leak-off flow through the standpipe has increase on at least one RCP.

D. A Residual Heat Removal Pump has been tested and the associated relief valve has lifted.

Answer: B

Explanation/Justification:

A. Incorrect. Plausible because PORVs and Safeties discharge to the PRT; however, the tailpipe temperatures have not increased.

B. Correct. PCV-135 discharges to the PRT.

C. Incorrect. Plausible because RCP seal return relief valve goes to the PRT; however, seal leakoff goes to the RCDT.

D. Incorrect. Plausible because a large portion of the system exists inside containment; however the relief valve is in the auixiliary building.

Technical

References:

9321-2738 Proposed References to be provided: None Learning Objective 12LP-ILO-RCSPZR 6 Question Source: Bank Question History: NA Question Cognitive Level: Comprehension 10 CRF Part 55 Content: 55.41 (b)7 Comments

Exam Outline Cross

Reference:

Level RO SRO Tier# 2 Group# 1 KIA# 007000K502 Knowledge of the operational implications of the following concepts as they apply to the PRTS: - Method of forming a steam bubble in the PZR Importance 3.1 3.4 Question # 33 Unit 2 is in Mode 5 performing steps to draw a bubble in the Pressurizer. The following steps have been completed:

  • Pressurizer is filled to an actual level of 50%
  • All Pressurizer heaters are energized
  • Pressure is at atmospheric pressure Per procedure, the PORVs will be closed after pressure reaches 25 psi in the PRT.

Which of the following correctly describes the reason for keeping the PORVs open until this pressure is reached?

A. Required as part ensuring LTOPs is operable when initially pressurizing the RCS.

B. Provides a flowpath for venting non-condensable gases out of the Pressurizer.

C. Verification that the PORV tailpipe temperature device will respond to changes in temperature.

D. Maintains flow from the RCS into the Pressurizer to ensure boron concentrations are equalized.

Answer: 8

Explanation/Justification:

Original question Salem 1999 (INPO QID 15151)

Unit 1 is in Mode 5 performing steps to draw a bubble in the Pressurizer. The following steps have been completed:

- The Pressurizer is filled as indicated on the cold calibrated level channel

- All Pressurizer heaters have been energized

- Pressure is controlled at approximately 65 psig The next major action is to manually open PR 1 & PR2 for 10-15 minutes when the Pressurizer reaches approximately 300 degrees F.

Which of the following correctly describes the reason for opening PR 1 & PR2?

Answer:

Provides a flowpath for venting non-condensable gases out of the Pressurizer Dist 1:

Required as part of the operability check for PR1 & PR2.

Dist 2:

Verification that the PORV tailpipe temperature device will respond to changes in temperature.

Dist 3:

Establishes flow from the RCS into the Pressurizer to ensure boron concentrations are equalized.

Explanation:

Per SOP-3.3 Section 4.1 0, the Pressurizer is vented to the Vent Header by keeping the PORVs open until pressure reaches 25# in the PRT.

A. Incorrect but plausible. Slight change from original question because the valves are not cycled as they apparently are at Salem. It is plausible because it is important that LTOPs be operable in this condition.

B. Correct answer.

C. Incorrect but plausible because this would in fact provide an opportunity to operationally check tailpipe temperature response.

D. Incorrect but plausible because having spray valves open does this. Also, opening a PORV under operating conditions can lead to flow from RCS to PZR.

Technical

References:

2-SOP-3.3 Proposed References to be provided: None Learning Objective 12LP-ILO-RCSPZR 11 Question Source: Bank Question History: Salem 1999 Question Cognitive Level: Comprehension 10 CRF Part 55 Content: 55.41 (b) 10 Comments

Exam Outline Cross

Reference:

Level RO SRO Tier# 2 Group# 1 KIA# 008000K301 Knowledge of the effect that a loss or malfunction of the CCWS will have on the following: - Loads cooled by CCWS Importance 3.4 3.5 Question # 34 The plant is operating at 100% power. During I&C testing, a component caused a Phase B containment isolation signal. What affect does this have on the operating Reactor Coolant Pumps per AOP-CCW-1?

A. All RCP cooling by CCW is lost. Continued operation of the RCPs is allowed as long as seal injection remains in service and bearing temperatures remain below 200°F.

B. All RCP cooling by CCW is lost. The RCPs must be tripped within 2 minutes if the Phase B can not be reset.

C. Only RCP Thermal Barrier cooling by CCW is lost. Continued operation of the RCPs is allowed as long as seal injection remains in service.

D. Only RCP Motor cooling by CCW is lost. The RCPs must be tripped within 2 minutes if the Phase B can not be reset.

Answer: B

Explanation/Justification:

A Phase B signal will cause a loss of all CCW to RCPs. With charging in service, seal cooling will be maintained. However, motor cooling will be completely lost requiring the RCPs to be tripped regardless of charging pump status.

A. This choice is not correct because AOP-CCW-1 directs tripping RCPs if all CCW cooling is lost OR if bearing temperatures exceed 2000F. It is plausible that a candidate could think that both conditions are needed.

B. Correct answer.

C. Incorrect. Plausible if a candidate has a misconception as to what cooling is lost on a Phase 8 signal. The second part of the choice would be correct if the first part was how the system is designed.

D. Incorrect. Plausible if a candidate has a misconception as to what cooling is lost on a Phase 8 signal. The second part of the choice would be correct if the first part was how the system is designed.

Technical

References:

2-AOP-CCW-1 3-AOP-CCW-1 Proposed References to be provided: None Learning Objective 12LP-ILO-CCW001 5 Question Source: New Question History: None Question Cognitive Level: Fundamental Knowledge 10 CRF Part 55 Content: 55.41 (b) 4 Comments

Exam Outline Cross

Reference:

Level RO SRO Tier# 2 Group# 1 KIA# 010000K601 Knowledge of the effect of a loss or malfunction of the following will have on the PZR PCS: -

Pressure detection systems Importance 2.7 3.1 Question # 35 Given the following:

  • Pressurizer Pressure Control is selected to the normal alignment (OFT CH Ill IV position).
  • Actual pressurizer pressure is 2240 psig and rising.
  • The modulating heaters are energized.
  • The bellows for channel 1 pressurizer pressure instrument (PT-455) is stuck with a constant output of 2215 psig.

Which ONE of the following describes the response of the pressure control system if the operator takes NO further action?

A. Pressure will rise until one PORV opens to control pressure.

B. Pressure will rise until one spray valves opens to control pressure.

C. Pressure will rise until the high pressure Reactor Trip setpoint is reached and the Reactor will trip.

D. Pressure will cycle on the modulating heaters at a higher setpoint.

Answer: A

Explanation/Justification:

Significantly modified version of 24900 Explanation:

Channel 1 is the controlling channel per the information given in this question.

With this failure, backup heaters will energize and spray will close because the control system will continue to sense low pressure. One PORV is controlled by the controlling channel and will never open in automatic. However, the other PORV will open when two of the other PZR pressure channels reach the setpoint. The high pressure trip setpoint will never be reached.

A. Correct answer B. Incorrect but plausible. It is plausible that the spray valve could be paired up with individual instrument channels.

C. Incorrect but plausible. Each PORV have a "confirmation" channel. There is a control setting/failure configuration that will lead to having no PORV open automatically. However, that is not true for the given configuration.

D. Incorrect but plausible. A candidate could believe that for a failure this close to the normal pressure setpoint, that pressure could cycle on the modulating heaters.

Technical

References:

2-AOP-INST-1 Proposed References to be provided: None Learning Objective 12LP-ILO-AOPINT 8 Question Source: Modified Question History: NA Question Cognitive Level: Comprehension 10 CRF Part 55 Content: 55.41 (b) 5 Comments

Exam Outline Cross

Reference:

Level RO SRO Tier# 2 Group# 1 KIA# 0130002130 Conduct of Operations -Ability to locate and operate components, including local controls.

Importance 4.4 4 Question # 36 An inadvertent Safety Injection Actuation (SIA) and automatic Reactor Trip has occurred. It was noted during the performance of E-0, Reactor Trip or Safety Injection, that Instrument Air to Containment Isolation Valve, PCV-1228 failed to automatically close as required, and had to be manually closed. Sl has subsequently been placed in DEFEAT and has been RESET in E-0. However, Containment Isolation (CIA) Phase A could NOT be reset when attempted.

Which of the following could be a cause for the failure of CIA, Phase A to reset?

A. Instrument Air to Containment Isolation Valve, PCV-1228 failed to AUTOMATICALLY close.

B. Control switch for the Weld Channel & Penetration Pressurization System valves (WCPPS) is OPEN.

C. Equipment Hatch Solenoid control switch is in INCIDENT.

D. Isolation Valve Seal Water Valve, 1410 control switch is in CLOSE.

Answer: D

Explanation/Justification:

A. Incorrect. Plausible because the switch position of 1228 is an input to Phase A reset circuit; however, the valve position is not.

B. Incorrect. Plausible because most phase A valves are closed when a signal is generated; however, WCPPS valves are opened and the switch must be in OPEN.

C. Incorrect. Plausible because the Equipment Hatch Solenoids are inputs to the Phase A reset circuit; however INCIDENT is the correct postion.

D. Correct. 1410 is Opened by a Phase A signal and the switch must be in OPEN to reset Phase A.

Technical

References:

2-E-0 Proposed References to be provided: None Learning Objective 12LP-ILO-ESS001 5 Question Source: Bank Question History: NA Question Cognitive Level: Comprehension 10 CRF Part 55 Content: 55.41 (b) 7 Comments

Exam Outline Cross

Reference:

Level RO SRO Tier# 2 Group# 1 KIA# 012000K304 Knowledge of the effect that a loss or malfunction of the RPS will have on the following:-

ESFAS Importance 3.8 4.1 Question # 37 The unit was online at 15% power during power ascention. 21 RCP tripped.

Attempts to trip the reactor from the control room were unsuccessful. The NPO tripped the reactor by opening the rod drive MG set output breakers.

Which of the following is a consequence of these conditions?

A. Feedwater isolation signal cannot be reset without using the key defeat switches.

B. MOV BFD-5 though 5-3 and 90 through 90-3 will not close on a feedwater isolation signal.

C. Low Flow Bypass Feed Regulating Valves will not close on low temperature D. Motor driven AFW pumps will not automatically start if area wide blackout occurred.

Answer: C

Explanation/Justification:

A. Incorrect. Plausible because the key defeat switches are used for Safety Injections and Phase A isolation, but they are not used in feedwater isolation. If it is necessary to override feedwater isolation, it is done by lifting a lead as in AOP-SSD-1.

B. Incorrect. Plausible because there are two categories of feedwater isolation.

One signal (low Tavg with Reactor Trip) only closes main feed regulating and low flow bypass valves. The second signal (High SG level or Safety Injection) closes MFRV, LFBV, Main Boiler Feed Pump discharge valve, motor operated isolation valves and trips the MBFP. Reactor trip breaker position will not impact the MOV closure during a FWIS.

C. Correct. At 15% power the MFRVs would be closed and the LFBVs would be in service. With both reactor trip breakers closed the signal to close the LFBV (and MFRV if they were open) will not occur.

D. Incorrect. Plausible because the black out loads due not get a start signal unless there is a Unit Trip signal. The Unit Trip signal comes from a generator trip, but it is plausible that an operator could believe the reactor trip breakers provide this input.

Technical

References:

Logic Unit 3 Sheet 13 Proposed References to be provided: None Learning Objective 12LP-ILO-ESS001 5 Question Source: New Question History: NA Question Cognitive Level: Fundamental Knowledge 10 CRF Part 55 Content: 55.41 (b) 5 Comments

Exam Outline Cross

Reference:

Level RO SRO Tier# 2 Group# 1 KIA# 013000K201 Knowledge of bus power supplies to the following: -

ESFAS/safeguards equipment control Importance 3.6 3.8 Question # 38 Given the following conditions:

  • The plant is in Hot Shutdown.
  • 21 and 23 AFW pumps are in service feeding all four SGs.
  • One of the 125 VDC control power fuses to the 23 AFW breaker blows.

Which one of the following describes the effect on the operation of 23 AFW Pump?

A. 23 AFW Pump trips.

No indication or control is available from the Control Room.

B. 23 AFW Pump remains in service.

No indication or control is available from the Control Room.

All automatic trips are lost.

C. 23 AFW Pump remains in service.

No indication is available in the Control Room.

All pump trips are available.

D. 23 AFW Pump remains in service.

No indication or control is available from the Control Room.

Only overcurrent and manual trips at the breaker remain available.

Answer: D

Explanation/Justification:

This question is a significantly modified version of IPEC Bank 8719 Original Question:

Given the following conditions:

  • The plant is in Hot Shutdown.
  • 21 and 23 Auxiliary Boiler Feed Pumps (ABFPs) are in service feeding all four SGs.
  • The 125 VDC control power fuses to the 23 ABFP breaker become open (are blown).

Which one of the following describes the effect on the operation of 23 ABFP?

A. Breaker indication in CCR is lost CCR breaker control is lost Pump remains running B. Breaker indication in CCR is lost CCR breaker control is lost Pump will trip C. Breaker indication is available CCR breaker control is lost Pump will trip D. Breaker indication in CCR is available CCR breaker control is lost Pump will remain running Explanation:

The KIA states, "knowledge of bus power supplies to ... safeguards equipment control". This question is testing knowledge of bus power supply's effect on safeguards equipment control. This seemed to be an interpretation of this KIA that led to higher quality question as opposed to recognizing which power supply goes to a particular component.

There are two fuses in a fuse block at the breaker; either blowing causes a complete loss of control power. On a loss of control power, 480V breakers stay in their current position. Overcurrent protection remains available via the breaker

amptector device. Manually tripping the breaker via mechanical device on breaker is always available. All remote, electrical trips are lost. Control Room indication is lost when control power is lost.

A. Incorrect but plausible because a candidate could think that the breakers will open on loss of power like reactor trip breakers do. Also, there are low flow alarms associated with 21/23 AFW Pumps and U3 AFW Pumps actually trip on low flow. All of these make it plausible that a candidate would think the pump would trip. The second part of the distracter is correct.

B. Incorrect but plausible because this would be a true statement for some station 6.9KV breakers.

C. Incorrect but plausible because a candidate might think that the two fuses have separate functions. One fuse going to indication while the other fuse goes to control. All of the choices have no indication would actually support this misconception.

D. Correct answer Technical

References:

Syst Desc 27. 1 Proposed References to be provided: None Learning Objective 12LP-ILO-EDS03 7 Question Source: Modified Question History: NA Question Cognitive Level: Fundamental Knowledge 10 CRF Part 55 Content: 55.41 (b)7 Comments

Exam Outline Cross

Reference:

Level RO SRO Tier# 2 Group# 1 KIA# 0220002422 Emergency Procedures/Plan -

Knowledge of the bases for prioritizing safety functions during abnormal/emergency operations.

Importance 3.6 4.4 Question # 39 During a LOCA, emergency coolant recirculation capability was lost, and ECA-1.1, Loss of Emergency Coolant Recirculation, is currently in progress. A RED path is identified on the CONTAINMENT status tree, and transition to FR-Z.1, Response to High Containment Pressure, is performed.

What procedure should be used to operate the containment spray pumps, and why?

A. FR-Z.1 because it takes precedence over ECA-1.1.

B. FR-Z.1, because it provides for REDUCED containment spray.

C. ECA-1.1; it provides for REDUCED containment spray.

D. ECA-1.1; an ECA should be completed prior to transferring to a FR Procedure.

Answer: C

Explanation/Justification:

Duplicated from question no 17323 to make slight modification to choice B A. . Incorrect. Plausible because in general FR procedures have a higher priority than any other procedure.

B .. Incorrect. Plausible because FR-Z.1 has steps for lowering VC pressure by increasing cooling with fan cooler units.

C Correct. One of the goals of ECA-1.1 is to maintain core cooling from the RWST. Reducing spray flow will aid in this goal.

D Incorrect. Plausible because the procedure referenced is correct; however the reason is not correct Technical

References:

2-ECA-1.1 2-ECA-1.1 BG 2-FR-Z.1 2-FR-Z.1 BG Proposed References to be provided: None Learning Objective 12LP-ILO-EOPC12 6 Question Source: New Question History: Unit 2 NRC 2008 Question Cognitive Level: Fundamental Knowledge 10 CRF Part 55 Content: 55.41 (b) 10 Comments

Exam Outline Cross

Reference:

Level RO SRO Tier# 2 Group# 1 KIA# 022000A301 Ability to monitor automatic operation of the CCS, including: -

Initiation of safeguards mode of operation Importance 4.1 4.3 Question # 40 The following conditions exist at Unit 2:

  • Plant heatup was in progress at the end of an outage
  • RCS temperature was 450°F
  • Five Containment Fan Cooler Units (FCUs) were in service
  • All other systems were in expected alignment for these conditions
  • No personnel were in containment
  • A loss of RCS coolant occurred which caused an automatic safety injection
  • 480V Bus 3A de-energized with fault indications during the transient
  • Sl was reset Assuming no other operator actions, what is the configuration of Containment Cooling at this time?

A. Three FCUs are in service, valves 1104 and 1105 are SHUT.

B. Three FCUs are in service, valves 1104 and 1105 are OPEN.

C. Four FCUs are in service, valves 1104 and 1105 are SHUT.

D. Four FCUs are in service, valves 1104 and 1105 are OPEN.

Answer: D

Explanation/Justification:

11 04 and 1005 are normally shut service water valves that get an open signal on Sl. They remain open after Sl is reset. It is plausible that they would go closed on Sl reset (this is actually the case for Unit 3's 1004/1105 valves)

Bus 3A has one FCU, so four will be in service for these conditions. However, there a couple of reasons why having three in service is plausible. First, there is a 480V bus that supplies two FCUs, but it is not 3A. Second 3A and 2A get power from the same EDG, so a candidate could believe that the fault on 3A caused 2A to de-energize and/or strip.

Technical

References:

Logic Unit 2 Sheet 8 Proposed References to be provided: None Learning Objective 12LP-ILO-AOIRH1 8 Question Source: New Question History: NA Question Cognitive Level: Fundamental Knowledge 10 CRF Part 55 Content: 55.41 (b) 8 Comments

Exam Outline Cross

Reference:

Level RO SRO Tier# 2 Group# 1 KJA# 026000A203 Ability to (a) predict the impacts of the following malfunctions or operations on the CSS and (b) based on those predictions, use procedures to correct, control, or mitigate the consequences of those malfunctions or operations:

-Failure of ESF Importance 4.1 4.4 Question # 41 The following events happened at Unit 2:

  • The automatic spray setpoint was exceeded but spray failed to actuate
  • Sl was reset by procedure
  • The BOP has just initiated a manual spray Which of the following describes the effect of initiating manual spray?

A. An Sl signal will be generated. Safeguards equipment that was realigned following Sl reset will return to Sl configuration.

B. An Sl signal will be generated. Safeguards equipment that was realigned following Sl reset will not be affected by initiating manual spray.

C. An Sl signal will NOT be generated. Both Containment Spray Pumps will start, but since Sl was reset, the discharge MOVs will have to be opened.

D. An Sl signal will NOT be generated. Both Containment Spray Pumps will start and the discharge MOVs will open.

Answer: D

Explanation/Justification:

Manual spray does not cause an Sl, but automatic spray does. The signal will start both spray pumps and open both discharge MOVs.

A. Incorrect but plausible because if an Sl signal were generated some equipment would restart.

B. Incorrect but plausible because the second part of the answer is correct.

C. Incorrect but plausible because a candidate could believe that Sl is the signal that opens the valves and they went shut on reset.

D. Correct answer.

Technical

References:

Logic Unit 2 Sheet 8 Proposed References to be provided: None Learning Objective 12LP-ILO-ESS001 5 Question Source: New Question History: NA Question Cognitive Level: Fundamental Knowledge 10 CRF Part 55 Content: 55.41 (b) 8 Comments

Exam Outline Cross

Reference:

Level RO SRO Tier# 2 Group# 1 KIA# 039000A106 Ability to predict and/or monitor changes in parameters (to prevent exceeding design limits) associated with operating the MRSS controls including:- Main steam pressure Importance 3 3.1 Question # 42 Given:

  • The plant is at approximately 25% power.
  • 21 Main Boiler Feed Pump is in MANUAL.
  • 22 MBFP is tagged out.
  • PT -412A, Turbine First Stage Pressure failed during the previous shift and ALL actions of AOP-INST-1, Instrument/Controller Failures have been completed.

Assuming no operator action, which ONE of the following describes the effect of this failure?

A. Steam Dumps remain closed. RCS temperature rises until Atmospheric Dump valves open to control RCS temperature.

B. Steam Dumps remain closed. RCS temperature rises until Main Steam Safety Valves open to control RCS temperature.

C. Steam Dumps open. RCS temperature rapidly lowers until Main Steam Line Isolation and Safety Injection actuate.

D. Steam Dumps open. RCS temperature rapidly lowers until steam dumps close at 54rF. Steam dumps will then cycle to maintain 54rF.

Answer: C

Explanation/Justification:

Duplicated from question no 25162 altered from U3 question A. Incorrect but plausible if candidate has misconeption about steam dump operation.

B. Incorrect but plausible if candidate has misconeption about steam dump operation.

C. Correct answer. With 412A failed and actions of AOP-INST-1 complete, half the high steam flow logic is made up. When 404 fails the steam dumps will open and lower RCS Tave until the SI/Main Steam Isolation setpoint is reached.

D. Incorrect but plausible if candidate has misconeption about steam dump operation.

Technical

References:

Syst Desc 18.1 Proposed References to be provided: None Learning Objective 12LP-ILO-SDSHP 3 Question Source: Bank Question History: IP3 2010 Question Cognitive Level: Comprehension 10 CRF Part 55 Content: 55.41 (b) 7 Comments

Exam Outline Cross

Reference:

Level RO SRO Tier# 2 Group# 1 KIA# 059000K304 Knowledge of the effect that a loss or malfunction of the MFW System will have on the following:

- RCS Importance 3.6 3.8 Question # 43 Given:

  • The plant is operating at 15% power.
  • 21 MBFP is in service in Automatic
  • 22 MBFP is in standby.
  • Preparations are in progress to transfer from Low Flow Bypass Feed Regulating Valves (LFBV) to Main Feed Regulating Valves (MFRV).
  • PT-408A Main Boiler Feed Pump (MBFP) Discharge pressure failed low.

Which of the following describes the effect on the plant and what actions are necessary per the abnormal operating procedure?

RCS RCS Temp Pressure Action Take Manual Control 21 A. Decrease Decrease MBFP to control level Place 22 MBFP in B. Increase Increase Automatic Trip Reactor then Trip 21 C. Increase Increase MBFP Increase speed of 22 MBFP D. Decrease Decrease to control level Answer: A

Explanation/Justification:

A. Correct. Discharge pressure failing low will cause MBFP speed to increase.

Since LFBVs are in service, SG l;evel will increase. The increase in level will cause a decrease in pressure. Taking man control of the MBFP B. Incorrect. Plausible because if the MBFP slowed down this response would be correct. Placing 22 MBFP in auto might seem to correct the problem but it would not because it would slow down also.

C. Incorrect. Plausible because the MBFP speed would increase and SG level would increase; however, the RCS Temp and Pressure would not increase.

D. Incorrect. Plausible because RCS Temp and Pressure would decrease for a failure of 408A; however, MBFP speed and SG level will not decrease.

Technical

References:

2-AOP-INST-1 Proposed References to be provided: None Learning Objective 12LP-ILO-AOPINT 32 Question Source: New Question History: NA Question Cognitive Level: Comprehension 10 CRF Part 55 Content: 55.41 (b) 5 Comments

Exam Outline Cross

Reference:

Level RO SRO Tier# 2 Group# 1 KIA# 059000A401 Ability to manually operate and/or monitor in the control room: -

MFW turbine trip indication Importance 3.1 3.1 Question# 44 The following conditions exist at Unit 2:

Reactor power is 3% during startup 21 MBFP is in service Low Flow Bypass Feed Regulating Valves have just been placed in service Aux Feed Regulating Valves are all now shut SG NR Levels are all 48% and stable 22 MBFP Turbine is indicating a green light and red flag on the flight panel What actions (if any) are REQUIRED prior to securing 21 and 23 AFW Pumps per SOP-AFW-001?

A. No actions are required.

B. 22 MBFPT steam supply valves must be shut.

C. 22 MBFPT must be green flagged.

D. 22 MBFPT trip switch must be placed in pull-to-lock.

Answer: C

Explanation/Justification:

The red flag/green light configuration of 22 MBFPT switch shows a disagreement. This would be the indication if the turbine had tripped by a signal other than the switch. A switch/light disagreement is one of the start signals to 21 and 23 AFW pumps. Green flagging is a common Unit 2 term meaning to rotate the switch in the trip direction to make the switch indicator flag green.

A. Incorrect but plausible if candidate misunderstand either the indications or requirements for securing the pumps.

B. Incorrect but plausible if candidate does not understand the operation of the MBFPT.

C. Correct answer D. Incorrect but plausible if candidate does not understand the operation of the MBFPT switch.

Technical

References:

2-SOP-AFW-001 Proposed References to be provided: None Learning Objective 12LP-ILO-MFW001 9 Question Source: New Question History: NA Question Cognitive Level: Fundamental Knowledge 10 CRF Part 55 Content: 55.41 (b) 7 Comments

Exam Outline Cross

Reference:

Level RO SRO Tier# 2 Group# 1 KIA# 061000K402 Knowledge of AFW System design feature(s) and/or interlock(s) which provide for the following: - AFW automatic start upon loss of MFW pump, S/G level, blackout, or safety injection Importance 4.5 4.6 Question # 45 The following plant conditions exist:

  • A reactor trip has occurred coincident with a loss of off-site power
  • 22 EDG failed to auto start
  • 22 AFW Pump was tagged out for maintenance
  • NPO has been dispatched to investigate the failure of 22 EDG
  • 21 and 22 SG WR levels are 54% and lowering slowly
  • 23 and 24 SG NR levels are 12% and rising slowly The FSS and the NPO are ready to start 22 EDG. When the EDG is started and the bus is energized, the motor driven AFW pump will:

A. Not automatically start because the pump 86 relay must be reset.

B. Automatically start without any time delay because SG levels are less than 10% in 2/4 SGs.

C. Not automatically start because the pump was placed in trip pull out.

D. Automatically starts after a time delay when blackout loads are sequenced on.

Answer: D

Exp Ia nation/Justification:

A. Incorrect. Plausible because an 86 lockout feature exists for CCW and other pumps to prevent start on low pressure; however, there is no 86 for AFW.

B. Incorrect. Plausible because candidate must remember that sequence timer starts when the bus is re-energized not from the time of the incident.

C. Incorrect. Plausible because some pumps are procedurally placed in trip pull out to prevent an automatic start. AFW is only placed in trip pull out during loss of all AC power.

D. Correct.

Technical

References:

2-E-0 2-E-0 BG Proposed References to be provided: None Learning Objective 12LP-ILO-MFW001 9 Question Source: Bank Question History: NA Question Cognitive Level: Comprehension 10 CRF Part 55 Content: 55.41 (b)?

Comments

Exam Outline Cross

Reference:

Level RO SRO Tier# 2 Group# 1 KIA# 062000A204 Ability to (a) predict the impacts of the following malfunctions or operations on the A. C.

Distribution System and (b) based on those predictions, use procedures to correct, control, or mitigate the consequences of those malfunctions or operations:

- Effect on plant of de-energizing a bus Importance 3.4 3.1 Question # 46 The unit is being restarted following a weekend outage.

The watch team has just reset the Main Generator 86P and 86 BU relays in preparation for synchronizing the Main Generator.

6.9KV Bus 5 is faulted.

Which one of the following will be the resultant 480V electrical alignment and what procedure should be entered?

A. Bus 2A is energized from 22 EDG. Bus 5A is de-energized, 21 EDG is running. The crew should enter 2-AOP-480V-1, Loss of Normal Power to Any 480V Bus.

B. Bus 2A is de-energized, 22 EDG is running. Bus 5A is energized from 21 EDG. The crew should enter 2-AOP-480V-1, Loss of Normal Power to Any 480V Bus.

C. Bus 2A and 5A are energized from 21 and 22 EDGs respectively. The crew should enter 2-AOP-138kV-1, Loss of Power to 6.9KV Bus 5 And/Or 6.

D. Bus 2A and 5A are de-energized. All EDGs are running. The crew should enter 2-AOP-138kV-1, Loss of Power to 6.9KV Bus 5 And/Or 6.

Answer: D Explanation/Justification:

A. Incorrect. Plausible because bus 2A is supplied from 22 EDG and would loose power under the above conditions; however, the EDG breaker would not close due to reset of 86P and BU. It is correct that bus 5A would be de-energized.

AOP-480V-1 would correct; if only 5A was de-energized.

B. Incorrect. Plausible because bus 5A is supplied from 21 EDG and would loose power under the above conditions; however, the EDG breaker would not close due to reset of 86P and BU. It is correct that bus 2A would be de-energized.

C. Incorrect. Plausible because this would be true if 86P and BU had not been reset. AOP-138KV-1 is correct for these condtions.

D. Correct. While the EDGs will start, the conditions to close the EDG breakers will not be satisfied. The requirement for a "unit trip" is from 86P and BU. AOP-138KV-1 is correct for these condtions.

Technical

References:

Syst Desc 27.1 Proposed References to be provided: None Learning Objective 12LP-ILO-EDSEDG 2 12LP-ILO-EDS08 3 Question Source: Bank Question History: NA Question Cognitive Level: Comprehension 10 CRF Part 55 Content: 55.41 (b)6 Comments

Exam Outline Cross

Reference:

Level RO SRO Tier# 2 Group# 1 KIA# 063000K201 Knowledge of bus power supplies to the following: - Major DC loads Importance 2.9 3.1 Question# 47 The crew is performing actions in 2-AOP-SSD-1 due to a fire in the 480V switchgear room. AII480V buses are de-energized and power is being supplied by the Appendix R diesel generator. The procedure directs actions to purge the generator with C02 if available.

What is the reason for purging the generator?

A. Allows breaking vacuum on the turbine/generator.

B. Allow securing Emergency Seal Oil Pump.

C. More rapidly slow down the turbine/generator.

D. Remove hydrogen to allow access to generator disconnect links.

Answer: B

Explanation/Justification:

A. Incorrect. Plausible because breaking vacuum is done.

B. Correct. Securing major DC loads extends time batteries can supply DC power to Instrument buses and other equipment.

C. Incorrect. Plausible because slowing down the turbine/generator will allow securing the Emergency Oil Pump; however these actions are not designed to slow the generator faster.

D. Incorrect. Plausible because power to 6.9 KV buses could be supplied by back feeding via the Main Transformers and Unit Aux.

Technical

References:

2-AOP-SSD-1 Proposed References to be provided: None Learning Objective 12LP-ILO-ASSD 11 Question Source: New Question History: NA Question Cognitive Level: Comprehension 10 CRF Part 55 Content: 55.41 (b) 10 Comments

Exam Outline Cross

Reference:

Level RO SRO Tier# 2 Group# 1 KIA# 062000A407 Ability to manually operate and/or monitor in the control room: -

Synchronizing and paralleling of different ac supplies Importance 3.1 3.1 Question # 48 Unit 2 is preparing to synchronize to the grid with the following conditions:

  • Main Transformer Secondary voltage and System voltage are indicating the same value.
  • The synchroscope is rotating slowly in the counter-clockwise direction.
  • All other conditions to synchronize have been met.

Which one of the following sets of actions is required per the turbine generator startup SOP prior to closing the selected output breaker?

A. AC Regulator operated in the RAISE position to establish output voltage 8-20 KV greater than system voltage.

Governor Control operated in the RAISE position to enable synchronization.

B. AC Regulator operated in the LOWER position to establish output voltage 8-20 KV less than system voltage.

Governor Control operated in the RAISE position to enable synchronization.

C. Governor Control operated in the LOWER position to enable synchronization.

AC Regulator operated as necessary to maintain voltages matched.

D. Governor Control operated in the RAISE position to enable synchronization.

AC Regulator operated as necessary to maintain voltages matched.

Answer: A Explanation/Justification:

Explanation:

2-SOP-26.4 has a caution that specifies that output voltage should be 8-20 KV higher than system voltage at synchronization. This is not a typical requirement when synchronizing (typically voltage is matched). The question was not asked with a choice of matched because this would be too trivial of a point to know from memory. However, given a choice of slightly higher or lower voltage, a well prepared operator should remember this from startup training. Either of these two conditions is plausible since it would just mean slightly leading vs. lagging VARs when the output breaker is closed. The operator is going to have to raise frequency to establish the synch scope rotating slowly in the clock-wise direction.

It is plausible that a candidate may not remember if this is lower or raise. Adding to the plausibility of both choices is that the voltage adjustment will affect frequency. It will actually make a larger adjustment needed in the raise direction.

"A" is the correct answer. 8 and C are half correct and D is all incorrect.

Technical

References:

2-SOP-26.4 Proposed References to be provided: None Learning Objective 12LP-ILO-POP007 4 Question Source: Modified Question History: Sequoyah NRC 2010 Question Cognitive Level: Fundamental Knowledge 10 CRF Part 55 Content: 55.41 (b) 7 Comments

Exam Outline Cross

Reference:

Level RO SRO Tier# 2 Group# 1 KIA# 064000K411 Knowledge of ED/G System design feature(s) and/or interlock(s) which provide for the following:- Automatic load sequencer: safeguards Importance 3.5 4 Question# 49 Given:

  • The plant was operating at 100% power
  • A large steam break accident occurred on the west header.
  • All systems functioned as designed.

After approximately 5 minutes a drop in offsite system voltage causes 480V Bus 5A voltage to drop to 41 OV. System voltage is subsequently restored to normal one minute later.

Which of the following describes the effect on Bus 5A and its loads?

A. Load breakers on 5A for non-safeguards equipment open, but the bus stays energized. Safeguards loads will remain energized.

B. Load breakers on 5A for safeguards and non-safeguards equipment open, but the bus stays energized. Safeguards equipment must be manually restarted.

C. The normal supply breaker to Bus 5A opens, load breakers for safeguards and non-safeguards equipment open, and power to the bus is restored by 21 EDG. Safeguards loads will be sequenced on the bus.

D. The normal supply breaker to Bus 5A opens, load breakers for non-safeguards equipment open, and power to the bus is restored by 21 EDG.

Safeguards loads will remain energized.

Answer: C

Explanation/Justification:

A. Incorrect. Plausible because removing non-essential loads may assist in raising bus voltage, however, sustained undervoltage for greater than 9 seconds will result in opening the bus supply breaker.

B. Incorrect. Plausible because this is true if this condition occurred after Sl was reset.

C. Correct. After 9 seconds < 420 V with an Sl signal, the bus normal feed breaker will open. With 21 EDG already running the output breaker will close re-energizing the bus. All loads will sequence on according to their timers.

D. Incorrect. Plausible because the normal feed breaker will open after approximately 9 seconds; however all loads will strip and sequence back on when the EDG breaker closes.

Technical

References:

2-ARP-FCF Proposed References to be provided: None Learning Objective 12LP-ILO-EDS01 3 12LP-ILO-EDS01 9 Question Source: New Question History: NA Question Cognitive Level: Comprehension 10 CRF Part 55 Content: 55.41 (b) 7 Comments

Exam Outline Cross

Reference:

Level RO SRO Tier# 2 Group# 1 KIA# 073000K101 Knowledge of the physical connections and/or cause-effect relationships between the PRM System and the following systems: - Those systems served by PRMs Importance 3.6 3.9 Question # 50 Given:

  • Unit 2 is at 60% power.
  • The 22 SG has developed a 60 gpd tube leak.
  • Operators are preparing to shutdown the Unit due to fuel failure resulting in RCS activity increasing at a steady rate.

Based on these plant conditions, which ONE of the following identifies how R-45 (Steam Jet Air Ejector) and R-61 B (22 SG N-16) will respond as RCS activity increases?

A. R-45 indication will increase; R-61 B indication will increase.

B. R-45 indication will increase; R-61 B indication will not change.

C. R-45 indication will not change; R-61 B indication will increase.

D. R-45 indication will not change; R-61 B indication will not change.

Answer: B

Explanation/Justification:

A. Incorrect. First part is correct as R-45 is sensitive to changes in Ar-41 activity and thus will increase as RCS activity increases. Second part is incorrect but plausible because the applicant who is not aware that N-16 monitor is NOT sensitive to changes in RCS activity may conclude that this distractor makes logic sense because activity is increasing.

B. Correct. First part is correct as discussed in Distractor A. Second part is also correct as mentioned above, the N-16 monitors are not sensitive to changes in RCS activity.

C. Incorrect. Plausible as discussed above, since the applicant may reverse which monitor is sensitive to RCS activity change and which is not.

D. Incorrect. First part is incorrect as described in Distractor A, R-45 monitor is sensitive to RCS activity changes and thus will show an increasing trend.

Second part is correct as discussed in Distractor B (monitor is not sensitive to changes in RCS activity).

Technical

References:

Proposed References to be provided: None Learning Objective 12LP-ILO-AOPSG1 10 12LP-ILO-AOPSG1 9 Question Source: Bank Question History: NA Question Cognitive Level: Comprehension 10 CRF Part 55 Content: 55.41 (b) 5 Comments

Exam Outline Cross

Reference:

Level RO SRO Tier# 2 Group# 1 KIA# 061000K501 Knowledge of the operational implications of the following concepts as they apply to the AFW System:- Relationship between AFW flow and RCS heat transfer Importance 3.6 3.9 Question# 51 A fault on 480V Bus 3A occurred and operators initiated a manual reactor trip from 100% power. All equipment responded normally for these conditions. After verifying that Sl is NOT required in E-0, Reactor Trip or Safety Injection, how is AFW status addressed?

A. No action is taken in E-0 because there will be >400 gpm AFW flow.

B. No action is taken in E-0 because there will be >760 gpm AFW flow.

C. 22 AFW pump will be used to feed 23 and 24 SGs because E-0 Step 4 attempts to establish >760 gpm AFW flow if Sl is NOT required.

D. 22 AFW pump will be used to feed 21 and 22 SGs because E-0 Step 4 attempts to establish >760 gpm AFW flow if Sl is NOT required.

Answer: D Explanation/Justification:

Duplicated from question no 25195. Modified for use at IP2 KA match because the required flow rates are based on assume RCS heat rates for given conditions.

A. Incorrect but plausible since heat sink is satisfied.

B. Incorrect but plausible if a candidate makes mistake on power supplies.

C. Incorrect but plausible if a candidate makes mistake on which AFW pump is lost.

D. Correct. E-0 attempts to establish greater 760 gpm.

Technical

References:

2-E-0 Proposed References to be provided: None Learning Objective 12LP-ILO-EOPEOO 9 Question Source: Bank Question History: IP3 2010 Question Cognitive Level: Comprehension 10 CRF Part 55 Content: 55.41 (b) 10 Comments

Exam Outline Cross

Reference:

Level RO SRO Tier# 2 Group# 1 KIA# 076000A201 Ability to (a) predict the impacts of the following malfunctions or operations on the SWS and (b) based on those predictions, use procedures to correct, control, or mitigate the consequences of those malfunctions or operations:

-Loss of SWS Importance 3.5 3.7 Question # 52 The following plant conditions exist:

  • Reactor power is 15%
  • Main Generator load is 112 MWe
  • 11 RWP is tagged out for maintenance
  • 12 RWP just tripped Based upon the plant conditions the operators should:

A. Trip the reactor, trip the turbine, go to E-0, (Reactor Trip or Safety Injection)

B. Trip the turbine, go to AOP-TURB-1, (Main Turbine Trip Without a Reactor Trip)

C. Initiate SOP 24.1, (Service Water System Operation) to swap to two header operation D. Initiate POP 3.1, (Plant Shutdown From 45% Power); reduce Turbine load if increasing turbine/generator temperatures observed.

Answer: B

Explanation/Justification:

A. Incorrect. Plausible because this would be correct if power was > P-8 (19%).

B. Correct.

C. Incorrect. Plausible because this would be correct if the turbine was not on the line.

D. Incorrect. Plausible because this is the correct procedure to perform a normal plant shutdown.

Technical

References:

2-AOP-SW-1 Proposed References to be provided: None Learning Objective 12LP-ILO-AOPSW1 3 Question Source: Bank Question History: NA Question Cognitive Level: Comprehension 10 CRF Part 55 Content: 55.41 (b) 5 Comments

Exam Outline Cross

Reference:

Level RO SRO Tier# 2 Group# 1 K/A# 078000K402 Knowledge of lAS design feature(s) and/or interlock(s) which provide for the following:-

Cross-over to other air systems Importance 3.2 3.5 Question # 53 Normal plant alignment of the air systems exist. 11 Centac has tripped, Station air pressure is 110 psig and declining. Which of the following best describes the automatic start sequence as pressure continues to decline? (No operator action)

A. 12 Centac, 21 Station Air compressor, 21 and 22 Instrument Air compressors, Station Air back up to WCCPPS, Nitrogen backup to WCCPPS B. 21 Station Air compressor, 21 and 22 Instrument Air compressors, Station Air back up to WCCPPS, Nitrogen backup to WCCPPS C. 21 and 22 Instrument Air compressors, 21 Station Air compressor, Station Air back up to WCCPPS, Nitrogen backup to WCCPPS D. 21 Station Air compressor, 21 and 22 Instrument Air compressors, Nitrogen back up to WCCPPS, Station Air backup to WCCPPS Answer: B

Exp Ia nation/Justification:

A. Incorrect. Plausible because station air and instrument air compressor starts are in the correct sequence, but 12 Centac would not auto start.

B. Correct. Station Air Compressor starts at 112 psig, and Instrument air compressor starts at 90 psig.

C. Incorrect. Plausible because Nitrogen backup and Station Air to WCCPPS are correct, but Instrument air compressor and station air compressor are swapped.

D. Incorrect. Plausible because station air instrument air compressor starts are correct, but eh nitrogen backup and station air to WCCPPS is swapped.

Technical

References:

Syst Desc 29.2 Syst Desc 29.3 Proposed References to be provided: None Learning Objective 12LP-ILO-SA01 7 Question Source: Bank Question History: NA Question Cognitive Level: Fundamental Knowledge 10 CRF Part 55 Content: 55.41 (b)7 Comments

Exam Outline Cross

Reference:

Level RO SRO Tier# 2 Group# 1 KIA# 103000A403 Ability to manually operate and/or monitor in the control room: -

ESF slave relays Importance 2. 7 2.7 Question # 54 Given:

  • Plant is at 100% power.
  • 22 Instrument bus has tripped due to a fault.
  • The crew is performing actions in 2-AOP-IB-1.
  • The procedure directs placing the TOPAZ inverter in service.

What is the reason for placing the TOPAZ inverter in service?

A. Provide power to operate the Containment Spray Master relay B. Provide power to operate the Containment Spray Logic relays.

C. Provide power to Bistable Status Panel on Panel SOF D. Provide power to Intermediate Range Nl Rx Trip and Rod Stop slave relays Answer: B

Exp Ia nation/Justification:

A. Incorrect. Plausible because the TOPAZ inverter is used to supply containment spray slave relays (energize to actuate).

B. Correct. Containment Spray logic is energize to actuate. The TOPAZ inverter provides power to allow the slave relays to be energized in the event of a Containment Spray Signal with a loss of IB 22.

C. Incorrect. Plausible because 22 Instrument bus provides power to the Bistable Status Panel; however it is not re-energized from the TOPAZ inverter.

D. Incorrect. Plausible because IB-22 provides power to N-36; however, when removing an IR channel blocking strips are place on relays that are expected to be energized.

Technical

References:

2-AOP-IB-1 Proposed References to be provided: None Learning Objective 12LP-ILO-AOPIB1 3 Question Source: New Question History: NA Question Cognitive Level: Fundamental Knowledge 10 CRF Part 55 Content: 55.41 (b) 7 Comments

Exam Outline Cross

Reference:

Level RO SRO Tier# 2 Group# 1 KIA# 103000A101 Ability to predict and/or monitor changes in parameters (to prevent exceeding design limits) associated with operating the Containment System controls including: - Containment pressure, temperature, and humidity Importance 3.7 4.1 Question # 55 Unit 2 is operating at 100% power. A small instrument air leak inside Containment causes a slow rise in Containment pressure. Containment pressure is currently 1.8 psig. In order to ensure that adequate margin to Containment Technical Specification pressure limits is maintained, which ONE of the following indicates the appropriate procedural action to reduce Containment pressure?

A. Maximize SW cooling to the Containment Fan Cooler Units.

B. The Containment should be vented using the Containment Purge System.

C. The Containment should be vented using the Containment Pressure Relief system.

D. All Containment Fan Cooler Units should be started or verified running.

Answer: C

Explanation/Justification:

A. Incorrect Increasing cooling (lowering temperature) may cause a slight pressure reduction but with continued in leakage pressure a release will have to be performed; B. Incorrect The Containment Purge system requires multiple reviews and sampling prior to use and is used only for shutdown conditions; C. Correct D. Incorrect Increasing cooling (lowering temperature) may cause a slight pressure reduction but with continued in leakage pressure a release will have to be performed.

Technical

References:

2-SOP-5.4.1 Tech Specs Proposed References to be provided: None Learning Objective 12LP-ILO-VCVCB 6 12LP-ILO-VCVCB 8 Question Source: Bank Question History: DC Cook NRC 2004 Question Cognitive Level: Fundamental Knowledge 10 CRF Part 55 Content: 55.41 (b) 5 Comments

Exam Outline Cross

Reference:

Level RO SRO Tier# 2 Group# 2 KIA# 001000K203 Knowledge of bus power supplies to the following:- One-line diagram of power supplies to logic circuits Importance 2.7 3.1 Question# 56 Given:

  • The unit is at 100% power
  • A fault occurred in the Buchanan switchyard resulting in a loss of 138KV power.

What is the immediate impact on the plant?

A The reactor will trip due to loss of both Rod Drive MG Sets when bus 138kV power is lost.

B. Since 21 Rod Drive MG set was required to be secured due to 22 EDG being OOS; the reactor trips when bus 138kV power is lost.

C. The reactor will remain at approximately 100% power with only 21 Rod Drive MG set energized.

D. The reactor will remain at approximately 100% power with both Rod Drive MG Sets energized.

Answer: C

Explanation/Justification:

Duplicated from question no 25961 A. Incorrect. Plausible because candidate my believe the rod drive MG sets are powered from 5A and 6A which both loose power when offsite power is lost.

Note: Buses 2A and 3A do not loose power and 21 Rod Drive MG Set will continue to operate (powered from 2A)

B. Incorrect. Plausible because a precaution in a 480V SOP states that If an EDG is the sole power supply for a 480V bus, and 2 rod drive MG sets are in operation, then MG set supplied by the running EDG should be shutdown.

Candidate may believe 21 RDMG set is already shutdown.

C. Correct.

D. Incorrect. Plausible because candidate may believe the MG set flywheel will maintain voltage until EDG re-energies the bus. The MG set supply breaker will trip on undervoltage.

Technical

References:

2-AOP-480V-1 Proposed References to be provided: None Learning Objective 12LP-ILO-ICROD 4 Question Source: Bank Question History: IP3 2013 Question Cognitive Level: Fundamental Knowledge 10 CRF Part 55 Content: 55.41 (b) 5 Comments

Exam Outline Cross

Reference:

Level RO SRO Tier# 2 Group# 2 KIA# 002000K607 Knowledge of the effect of a loss or malfunction on the following RCS components: - Pumps Importance 2.5 2.8 Question # 57 Given:

  • The Unit is operating at 5% power during a plant startup.
  • The System Operator notifies the control room that grid frequency is unstable What is the effect on the plant when grid frequency decreases to 57 Hz?

A. Frequency less than 57.5 Hz for greater than 0.3 seconds will cause a direct trip of the reactor at that time.

B. Frequency less than 57.5 Hz for greater than 30.5 seconds will cause a direct trip of the reactor at that time.

C. Frequency less than 57.5 Hz on any 2 inside 6.9KV busses will cause all RCP breakers to open. The reactor will NOT automatically trip on loss of flow.

D. Frequency less than 57.5 Hz on any 2 inside 6.9KV busses will cause all RCP breakers to open. The reactor will automatically trip on loss of flow.

Answer: C

Explanation/Justification:

9321-LL-31133 Sh 7 A. Incorrect. Plausible because Under Voltage not Underfrequency (75% of nominal value) for 0.3 seconds will trip the reactor. If the Voltage remains below setpoint for 30.5 seconds, the associated RCP will trip.

B. Incorrect. Plausible because Under Voltage not Under frequency (75% of nominal value) for 30.5 seconds will trip the associated RCP . If power is greater than 35% the reactor will trip on any single pump trip. If power is greater than 10% the reactor will trip if 2 of 4 6.9 kV buses exceed the setpoint and time.

C. Correct.

D. Incorrect since the reactor does not trip at this power.

Technical

References:

Syst Desc 28 Proposed References to be provided: None Learning Objective 12LP-ILO-ICRXP 8 13LP-ILO-RCSRCP 3 13LP-ILO-RCSRCP 4 Question Source: Bank Question History: Unit 3 NRC 2010 Question Cognitive Level: Fundamental Knowledge 10 CRF Part 55 Content: 55.41 (b) 7 Comments

Exam Outline Cross

Reference:

Level RO SRO Tier# 2 Group# 2 KIA# 014000K502 Knowledge of the operational implications of the following concepts as they apply to the RPIS: - RPIS independent of demand position Importance 2.8 3.3 Question # 58 Power is 88% with Control Bank D Demand Position is 198 Steps. All rods listed are in Control Bank D.

  • Control Rod F2 at 189 steps
  • Control Rod H8 at 223 steps
  • Control Rod B 10 at 185 steps

A. Only Control Rods H8 and K14 are misaligned because they have positive deviation > 12 steps.

B. Only Control Rod H8 is misaligned because it is ~ 24 steps from the demand position.

C. Only Control Rods K14 and H8 are misaligned because they are >16 steps from the demand position.

D. Only Control Rods B10, K14, and H8 are misaligned because they are

>12 steps from the demand position.

Answer: D

Explanation/Justification:

Duplicated from question no 23802. Modified for use on NRC exam As per Tech Specs, the maximum allowable deviation at this power level and rod height is 12 steps. However, choices are all plausible because there are different allowable differences depending on power level, rod height and direction of deviation.

A. Incorrect but plausible because there are different allowable differences based on whether deviation is negative or positive. The negative deviations are actually more limiting, but it is plausible that a candidate could confuse these.

B. Incorrect but plausible because, if power were <85%, this would be correct.

C. Incorrect but plausible because the spec is >16 steps at the normal100%

power rod height.

D. Correct Answer.

Technical

References:

Tech Specs Proposed References to be provided: None Learning Objective 12LP-ILO-ICROD 14 12LP-ILO-ICRPI14 Question Source: Bank Question History: None Question Cognitive Level: Comprehension 10 CRF Part 55 Content: 55.41 (b) 10 Comments

Exam Outline Cross

Reference:

Level RO SRO Tier# 2 Group# 2 KIA# 015000K403 Knowledge of NIS design feature(s) and/or interlock(s) which provide for the following: -

Reading of source range/intermediate range/power range outside control room Importance 3.9 4 Question # 59 The control room was evacuated and 2-AOP-SSD-1, Control Room Inaccessibility Safe Shutdown Control, was implemented due to a fire in the Control Room.

Which of the following describes where reactor power can be determined outside the control room?

A. Charging Pump Emergency Control Panel.

B. Fan House.

C. Electrical Tunnel.

D. Cable Spreading Room.

Answer: B

Exp Ia nation/Justification:

A. Incorrect. Plausible because source range indication may be desired at the charging station where boration is controlled.

B. Correct.

C. Incorrect. Plausible because the cables from the source range Nls penetrate the VC wall in the upper electrical tunnel.

D. Incorrect. Plausible because the reactor is tripped from the cable spreading room and local indication may be desired.

Technical

References:

2-AOP-SSD-1 Proposed References to be provided: None Learning Objective 12LP-ILO-ASSD 16 Question Source: New Question History: NA Question Cognitive Level: Fundamental Knowledge 10 CRF Part 55 Content: 55.41 (b) 6 Comments

Exam Outline Cross

Reference:

Level RO SRO Tier# 2 Group# 2 KIA# 017000A202 Ability to (a) predict the impacts of the following malfunctions or operations on the ITM System and (b) based on those predictions, use procedures to correct, control, or mitigate the consequences of those malfunctions or operations: -

Core damage Importance 3.6 4.1 Question # 60 Given:

  • The plant experienced a LOCA with core damage.
  • Containment Pressure is 8 psig and lowering.
  • RVLIS Natural Circulation Range is 57% and stable.

Which of the following is true regarding the individual Core Exit Thermocouple indications?

A. Accurate due to being located near the fuel assembly exit flow. The CRS must use indicated CET values for decision making.

B. Inaccurate due to high gamma fields as upper head voiding occurs. The CRS must use Adverse Containment values for decision making.

C. Accurate due to being calibrated at zero/low flow conditions. The CRS must use quadrant average calculated values for decision making D. Inaccurate due to steam bubble in the upper head area. The CRS must use program calculated CET values for decision making Answer: A

Explanation/Justification:

A. Correct: Thermocouples located on the core top plate.

B. Incorrect. Plausible because most instruments are affected by high gamma fields; however, thermocouples are not affected by high gamma fields. There are no adverse values for CET.

C. Incorrect. Thermocouples are calibrated for temperature response and their accuracy is not dependent on flow conditions.

D. Incorrect. Plausible because loop RTDs need flow to give a "representative" sample indication.

Technical

References:

Syst Desc 14 Proposed References to be provided: None Learning Objective 12LP-ILO-ICNXC 4 Question Source: New Question History: NA Question Cognitive Level: Comprehension 10 CRF Part 55 Content: 55.41 (b) 5 Comments

Exam Outline Cross

Reference:

Level RO SRO Tier# 2 Group# 2 KIA# 016000K309 Knowledge of the effect that a loss or malfunction of the NNIS will have on the following: -

ESFAS Importance 3.5 3.7 Question# 61 Steam Generator 21 "C" Pressure channel has failed low.

  • No actions have been taken.

Which one of the following additional failures will result in a Steamline Delta P Safety Injection?

A. Steam Generator 21 "A" Pressure channel failing high B. Steam Generator 22 "A" Pressure channel failing high C. Steam Generator 23 "B" Pressure channel failing low D. Steam Generator 24 "C" Pressure channel failing low Answer: B

Explanation/Justification:

Steam line Delta P Sl provides protection of a steam break accident upstream of MSIVs when they are closed. The signal is generated when one SG pressure is 155 psid lower than two of the remaining three SGs. The SG pressure channels are compared in a specific arrangement to generate this Sl signal. The initial failure results in 21 SG "appearing" more than 155 psi less than 24 SG. Any SG pressure transmitter failure (high or low) will generate a Delta P Sl input signal. It takes the proper arrangement of transmitters to actually result in a Safety Injection.

A Incorrect. This failure will make 22 SG "appear" > 155 psi lower than 21.

B. Correct. This failure will make 21 SG "appear" > 155 psi lower than 22 and since it already "appears that 21 SG is > 155 psi lower than 24 the Sl signal will occur.

C. Incorrect. This failure will make 23 SG "appear" > 155 psi lower than 24 D. Incorrect. This failure will make 24 SG "appear" equal to 21 (in the failed condition)

Technical

References:

Logic Unit 2 Sheet 12 Proposed References to be provided: None Learning Objective 12LP-ILO-ESS001 4 Question Source: Bank Question History: NA Question Cognitive Level: Fundamental Knowledge 10 CRF Part 55 Content: 55.41 (b) 7 Comments

Exam Outline Cross

Reference:

Level RO SRO Tier# 2 Group# 2 KIA# 029000A102 Ability to predict and/or monitor changes in parameters (to prevent exceeding design limits) associated with operating the Containment Purge System controls including: - Radiation levels Importance 3.4 3.4 Question # 62 The plant is in MODE 5 and in Day 4 of a Refueling Outage. Radiation monitor R-41, Vapor Containment Particulate Activity Monitor, has just gone into ALARM.

From the list below, select the actions that would occur from R-41 reaching the ALARM setpoint.

1. Containment pressure relief valves would receive a CLOSE signal.
2. Pressure Relief Fan would receive a TRIP signal.
3. PAB Exhaust Fan would receive a signal to divert through the Charcoal Filters.
4. CB Purge Fan would be sent a TRIP signal.
5. A containment Evacuation ALARM signal would be initiated.
6. Containment purge supply valves would receive a CLOSE signal.
7. RCV-014 would receive a CLOSE signal.
8. Containment purge exhaust valves would receive a CLOSE signal.

A. 1, 2, 3, 4, 6 B. 2, 3,5, 7,8 C. 1, 2,4, 5,6, 8 D. 2,3,4,6, 7, 8 Answer: C Explanation/Justification:

Items 3 and ?are plausible because they are correct for Radiation Monitor R-44 but not for R-41.

A. Incorrect B. Incorrect C. Correct D. Incorrect Technical

References:

2-SOP-12.3.3 Proposed References to be provided: None Learning Objective Question Source: Bank Question History: NA Question Cognitive Level: Fundamental Knowledge 10 CRF Part 55 Content: 55.41 (b) 11 Comments

Exam Outline Cross

Reference:

Level RO SRO Tier# 2 Group# 2 KIA# 045000A304 Ability to monitor automatic operation of the MT/G System, including: - T/G trip Importance 3.4 3.6 Question # 63 The unit is operating at 100% power You observe the white lights extinguished for

  • 20 AST Auto Stop Solenoid
  • 20 ASB Auto Stop Solenoid Which of the following pairs of turbine trips will be completely disabled by the above condition?

A. Low Vacuum and Manual pushbutton (flight panel)

B. High SG level and Thrust Bearing Wear C. Reactor Trip and AMSAC D. Low Bearing Oil Pressure and Safety Injection Answer: C

Explanation/Justification:

A. Incorrect. Plausible because Manual pushbutton will NOT function, but Low Vacuum will.

B. Incorrect. Plausible because High SG Level will NOT function, but Thrust Bearing Wear will.

C. Correct. Both are Solenoid Trips that require the solenoid valves to energize to open to cause a turbine trip.

D. Incorrect. Plausible because Safety Injection will NOT function, but Low Bearing Oil Pressure will.

Technical

References:

Syst Desc 28 Proposed References to be provided: None Learning Objective 12LP-ILO-ICRXP 4 12LP-ILO-ICRXP 9 Question Source: New Question History: NA Question Cognitive Level: Fundamental Knowledge 10 CRF Part 55 Content: 55.41 (b)7 Comments

Exam Outline Cross

Reference:

Level RO SRO Tier# 2 Group# 2 KIA# 072000A402 Ability to manually operate and/or monitor in the control room: -

Major components Importance 2.5 2.5 Question # 64 Which one of the following describes the MINIMUM requirement for automatic swap of CCR ventilation to the INCIDENT OUTSIDE AIR FILTERED FOR SI/HI RAD mode?

A. Alarm on R-1 CCR Area Radiation Monitor AND either R-38-1 or R-38-2 CCR Intake Radiation Monitors B. Alarm on R-1, CCR Area Radiation Monitor.

C. Alarm on either R-38-1 or R-38-2, CCR Intake Radiation Monitors.

D. Alarm on BOTH R-38-1 and R-38-2 CCR Intake Radiation Monitors Answer: C

Explanation/Justification:

A. Incorrect. Plausible because R-38-1 or R-38-2 will cause the ventilation system to shift to incident mode; however R-1 will not and it is not required to swap modes.

B. Incorrect. Plausible because it does monitor CCR Area Radiation and only one signal is necessary to swap to incident mode.

C. Correct. Either R-38-1 or R-38-2 in alarm will cause CCR ventilation to swap to incident mode.

D. Incorrect. Plausible because either R-38-1 or R-38-2 in alarm will cause CCR ventilation to swap to incident mode, but both are not required.

Technical

References:

2-SOP-12.3.3 Syst Desc 11 Proposed References to be provided: None Learning Objective 12LP-ILO-RMS001 3 12LP-ILO-RMS001 5 Question Source: Bank Question History: NA Question Cognitive Level: Fundamental Knowledge 10 CRF Part 55 Content: 55.41 (b) 11 Comments

Exam Outline Cross

Reference:

Level RO SRO Tier# 2 Group# 2 KIA# 086000K103 Knowledge of the physical connections and/or cause-effect relationships between the Fire Protection System and the following systems: - AFW System Importance 3.4 3.5 Question # 65 The following conditions exist at Unit 2:

  • A plant trip occurred four hours ago due to a seismic event
  • The only supply of AC power is the Appendix R Diesel Generator
  • RCS Temperature is 300°F
  • Verplanck fire trucks are on site to maintain fire hose pressure if needed
  • The CST tank lost inventory due to a crack at the bottom of the tank
  • City Water backup to AFW is not available In accordance with 2-SOP-ESP-001, Local Equipment Operation and Contingency Actions, which of the following would be a method of using fire water to feed steam generators?

A. Connect a fire hose to connection at suction to 21 AFW pump.

B. Connect a fire hose to connection at suction to 22 AFW pump.

C. Connect a fire hose to connection on line from CST to AFW.

D. Connect a fire hose to the SG chemical feed line.

Answer: D

Explanation/Justification:

The correct answer is D in accordance with 2-SOP-ESP-001 sections 4.8 or 4.9 depending on how fire water pressure is being maintained. Regardless of how pressure is being maintained, fire hose pressure will be around 100 psi and SG pressure at 300°F will be 52 psi.

2-SOP-ESP-001 also allows for connecting fire water to the city water supply of AFW, but this is precluded by the stem and not a choice. Another method not in 2-SOP-ESP-001 but in 858 procedures is to fill the CST from the top of the tank, but this is precluded by the stem and not a choice as well.

A Incorrect. Plausible because 21 AFW pump is the App R designated AFW pump, and Fire Water can be used to supply City Water which can then be used to supply the AFW system; however the flowpath described is not correct.

B. Incorrect. Plausible because 22 AFW pump is the App R designated AFW pump, and Fire Water can be used to supply City Water which can then be used to supply the AFW system; however the flowpath described is not correct.

C. Incorrect. Plausible because candidate may confuse the flowpath and method used to supply Fire Water to the AFW Pumps. The flowpath described is not correct.

D. Correct Technical

References:

2-SOP-ESP-001 Proposed References to be provided: None Learning Objective 12LP-ILO-MFW001 15 IOLP-ILO-AOPSEC 7 Question Source: New Question History: None Question Cognitive Level: Fundamental Knowledge 10 CRF Part 55 Content: 55.41 (b) 8 Comments

Exam Outline Cross

Reference:

Level RO SRO Tier# 3 Group#

KIA# 1940012123 Conduct of Operations- Ability to perform specific system and integrated plant procedures during all modes of plant operation.

Importance 4.3 4.4 Question# 66 The plant is in MODE 5. The RCS is operating with a water solid Pressurizer.

Which one of the following indications and actions confirms the pressurizer is water solid?

A. RCS pressure increasing with a decrease in letdown flow.

B. RCS pressure decreasing with a decrease in letdown flow.

C. RCS pressure stable with an increase in letdown flow.

D. RCS pressure stable with a decrease in letdown flow.

Answer: A

Exp Ia nation/Justification:

A. Correct B. Incorrect. Plausible because pressure would decrease with decreasing with increasing letdown flow.

C. Incorrect. Plausible because this indication may be present when collapsing or just drawing a pressurizer bubble, but this indication would be present with a bubble in the pressurizer.

D. Incorrect. Plausible because this indication may be present during collapsing of the pressurizer bubble, but this indication would be present with a bubble in the pressurizer.

Technical

References:

2-POP-3.3 3-POP-4.1 Proposed References to be provided: None Learning Objective 13LP-ILO-POP003 6 12LP-ILO-POP002 5 Question Source: Bank Question History: NA Question Cognitive Level: Comprehension 10 CRF Part 55 Content: 55.41 (b) 5 Comments

Exam Outline Cross

Reference:

Level RO SRO Tier# 3 Group#

KIA# 1940012140 Conduct of Operations-Knowledge of refueling administrative requirements.

Importance 2.8 3.9 Question # 67 When fuel is being transported between the VC and SFP, which ONE of the following describes the requirement for oversight of fuel handling operations?

A. Either a Reactor Engineer OR a fully qualified SRO may supervise Fuel Handling Activities in the VC OR SFP area.

B. A fully qualified SRO must be present for all fuel handling activities in BOTH the VC AND in the SFP area.

C. A fully qualified SRO must be present for all fuel handling activities in the VC ONLY.

D. A Reactor Engineer must be present for all fuel handling activities in the VC. A fully qualified SRO must be present for fuel handling activities in the SFP.

Answer: C

Explanation/Justification:

Duplicated from question no 23917 with some slight wording changes based on validation comments.

A. Incorrect. Plausible because a fully qualified SRO, however, the Reactor Engineer (Core Physics Monitor) is required to station in the control room not VC.

B. Incorrect. Plausible because the statement is true except for the requirement for the SRO to be present in the SFP.

C. Correct.

D. Incorrect. Plausible because a Reactor Engineer (Core Physics Monitor) is required to station in the control room not VC.

Technical

References:

Tech Specs Proposed References to be provided: None Learning Objective 12LP-ILO-FHD001 19 Question Source: Bank Question History: NA Question Cognitive Level: Fundamental Knowledge 10 CRF Part 55 Content: 55.41 (b) 10 Comments

Exam Outline Cross

Reference:

Level RO SRO Tier# 3 Group#

KIA# 1940012145 Conduct of Operations - Ability to identify and interpret diverse indications to validate the response of another indication.

Importance 4.3 4.3 Question # 68 A rapid load reduction has just been completed. You observe the following:

  • pressurizer sprays partially open
  • modulating heaters full off
  • backup heaters on Which of the following would cause these indications?

A. Controlling pressurizer pressure channel failed high B. Pressurizer level insurge from the downpower C. Controlling pressurizer level channel failed low D. Pressurizer program level decrease from the downpower Answer: 8

Explanation/Justification:

A. Incorrect. Plausible because spray valves would be open (fully) and modulating heaters would be off; however the backup heaters would be off also.

B. Correct. The insurge would cause pressure to increase, when actual pressurizer level exceeds program level by 5% the backup heaters would be energized.

C. Incorrect. Plausible because the control system would cause charging to increase which would cause actual pressurizer level to increase. Actual level increase may cause pressure to increase resulting in heaters off and sprays open. The backup heaters would be off.

D. Incorrect. Plausible because Pressurizer Program Level will decrease after the load reduction due to lowering Tavg; however, the indications are not consistent with this phase of the event.

Technical

References:

Proposed References to be provided: None Learning Objective 12LP-ILO-RCSPZR 4 13LP-ILO-RCSPZR 3 Question Source: Bank Question History: NA Question Cognitive Level: Fundamental Knowledge 10 CRF Part 55 Content: 55.41 (b) 5 Comments

Exam Outline Cross

Reference:

Level RO SRO Tier# 3 Group#

KIA# 1940012206 Equipment Control - Knowledge of the process for making changes to procedures.

Importance 3 3.6 Question # 69 In accordance with IP-SMM-AD-102, "IPEC Implementing Procedure Preparation, Review and Approval", which of the following would prevent changing a procedure using a temporary procedure change?

A. Editorial change to correct attachment numbers.

B. Changes are necessary due to a temporary plant condition.

C. A change to less restrictive acceptance criteria.

D. The procedure will be exited prior to completion.

Answer: C

Explanation/Justification:

There are 3 types of Temporary Procedure Changes, Advance, Conditional and Editorial. In addition, knowledge of "intent change" is necessary to answer this question.

A. Incorrect. Plausible because the change will become permanent and not just remain "temporary".

B. Incorrect. Plausible because an Advance Change TPC addresses conditions that will be made permanent. This specific condition is addressed by a Conditional TPC and is allowed.

C. Correct. This condition is specifically identified as an Intent Change not allowed by the administrative procedure.

D. Incorrect. Plausible because the necessity for a TPC may be questioned if the procedure will be exited before the actions of the TPC are completed.

Technical

References:

IP-SMM-AD-102 Proposed References to be provided: None Learning Objective IOWKB-ILO-ADMOO 2.2.6 Question Source: Bank Question History: NA Question Cognitive Level: Fundamental Knowledge 10 CRF Part 55 Content: 55.41 (b) 10 Comments

Exam Outline Cross

Reference:

Level RO SRO Tier# 3 Group#

KIA# 1940012212 Equipment Control - Knowledge of surveillance procedures.

Importance 3.7 4.1 Question # 70 April Sunday Monday Tuesday Wednesday Thursday Friday Saturday 1 2 3 4 5 6 7 8 9 10 11 12 13 14 15 16 17 18 19 20 21 A weekly surveillance test was completed at 1300 on Tuesday April 3rd.

In accordance with Technical Specifications, what is the LATEST this test can be scheduled to be completed this week, without exceeding an LCO?

A. 1900 on Tuesday April 1Oth B. 0500 on Wednesday April 11th C. 0700 on Thursday April 12th D. 0100 on Saturday April 14th Answer: C

Explanation/Justification:

The specified Frequency for each SR is met if the Surveillance is performed within 1.25 times the interval specified in the Frequency, as measured from the previous performance or as measured from the time a specified condition of the Frequency is met. The maximum frequency is 8.75 days or 8 days 18 hours2.083333e-4 days <br />0.005 hours <br />2.97619e-5 weeks <br />6.849e-6 months <br />.

A. Incorrect. Plausible because 6 hours6.944444e-5 days <br />0.00167 hours <br />9.920635e-6 weeks <br />2.283e-6 months <br /> is 25% of one day (24 hours2.777778e-4 days <br />0.00667 hours <br />3.968254e-5 weeks <br />9.132e-6 months <br />) not one week.

B. Incorrect. Plausible because 0500 on April 11th is approximately weekly frequency times 1.1 0.

C. Correct. This time is 1.25 times the required frequency.

D. Incorrect. Plausible because 0100 on April 14th is approximately weekly frequency times 1.50.

Technical

References:

Tech Specs Proposed References to be provided: None Learning Objective IOLP-ILO-ITS001 1 (3967)

Question Source: Bank Question History: Cooper Unit 1 NRC 2002 Question Cognitive Level: Comprehension 10 CRF Part 55 Content: 55.41 (b) 10 Comments

Exam Outline Cross

Reference:

Level RO SRO Tier# 3 Group#

KIA# 1940012305 Radiological Controls - Ability to use radiation monitoring systems, such as fixesd radiation monitors and alarms, portable survey instruments, personnel monitoring equipment, etc.

Importance 2.9 2.9 Question# 71 When a Digital Radiation Monitor Skid for a Liquid Process Radiation Monitor is purged, the reading on most skids after purge is not immediately reliable. What is one reason for this unreliability?

A. The detector components are cooled by the purge medium and need time to reach thermal equilibrium with the sampled process fluid.

B. The detector is electronically removed from service during purge and must be reactivated by local operation at the Skid's Local Control Unit (LCU).

C. All purge medium used is nitrogen and must be vented out of the detector's chamber before a reliable reading can be obtained.

D. The detector is purged by reverse flow of City Water. The detector is therefore reading a low radiation level fluid until it is refilled with the sample process fluid.

Answer: D

Explanation/Justification:

Question was modified for use on NRC Exam.

Explanation:

Per the lesson plan and system description, D is the correct answer.

A. Incorrect but plausible since temperature will have a very slight effect on a scintillation detector output. However, the detector itself is not cooled by the purge liquid, so this incorrect.

B Incorrect but it is plausible that a local reset could be required.

C. Incorrect but it is plausible that nitrogen would be used to purge the detector chamber.

D. Correct answer.

Technical

References:

Proposed References to be provided: None Learning Objective 12LP-ILO-RMS001 7 Question Source: Bank Question History: None Question Cognitive Level: Comprehension 10 CRF Part 55 Content: 55.41(b)11 Comments

Exam Outline Cross

Reference:

Level RO SRO Tier#

Group#

KIA# 1940012312 Radiological Controls -

Knowledge of radiological safety principles pertaining to licensed operator duties, such as containment entry requirements, fuel handling responsibilities, access to locked high-radiation areas, aligning filters, etc.

Importance 3.2 3.7 Question# 72 You are about to sign on to an RWP to operate a valve in the Reactor Coolant Filter Cell and notice there is no requirement to wear a respirator. You recall from your tour this morning that the cell is posted as a High Airborne Activity Area. You ask the HP tech about this and are told it is consistent with ALARA to NOT wear a respirator.

Why would NOT wearing a respirator in a High Airborne Activity Area be consistent with A LARA?

A. The respirator is only effective on particulates so there will be NO difference in TEDE.

B. The respirator will limit your vision and may be a safety hazard in "tight" spaces.

C. Wearing a respirator may increase your stay time and actually increase TEDE.

D. The Airborne Activity is short lived and the respirator will have NO impact on ALARA Answer: C

Explanation/Justification:

This question was previously used on Oyster Creek Unit 1 ILO exam. The only modification is that the job location was changed to the RCS Filter Cell. The question meets the KA since it tests the principle of TEDE/ALARA vs. absolute minimizing internal exposure. The distracters are plausible if the candidate knowledge radioactivity and radiological controls is weak.

Technical

References:

Proposed References to be provided: None Learning Objective IOWKB-ILO-ADMOO 1 Question Source: Bank Question History: Oyster Creek Unit 1 2005 Question Cognitive Level: Fundamental Knowledge 10 CRF Part 55 Content: 55.41 (b) 12 Comments

Exam Outline Cross

Reference:

Level RO SRO Tier# 3 Group#

KIA# 1940012435 Emergency Procedures/Plan -

Knowledge of local auxiliary operator tasks during an emergency and the resultant operational effects.

Importance 3.8 4 Question # 73 A reactor trip and loss of off-site power has occurred. All of the individual rod position indicators on the flight panel have failed to zero and all of the rod bottom lights are not lit, indicating a loss of power. What means are there for determining the actual rod position?

A. Energize Instrument Bus 24 from alternate source to reenergize analog meters B. Reset MCC- 24 to reenergize the individual rod position indication system.

C. Individual rod position indication is available using the plant computer PIGs.

D. Manually close bus 2A supply breaker from 22 EDG.

Answer: B

Explanation/Justification:

A. Incorrect. Plausible because candidate may believe that Instrument Bus 24 supplies power to the IRPI; however MCC-24 is the correct power supply.

B. Correct. MCC-24 is the correct power supply for the IRPis C. Incorrect. Plausible because PIGS can be used to check rod individual rod position; however, with MCC-24 de-energized PIGs will indicate 0.

D. Incorrect. Plausible because MCC-24 is powered from bus 2A and if bus 2A is de-energized, IRPis will not indicate; however, MCC-24 will strip on undervoltage and must be manually reset to re-energize IRPis.

Technical

References:

2-AOP-480V-1 Proposed References to be provided: None Learning Objective 12LP-ILO-ICRPI 7 Question Source: Bank Question History: NA Question Cognitive Level: Comprehension 10 CRF Part 55 Content: 55.41 (b)7 Comments

Exam Outline Cross

Reference:

Level RO SRO Tier# 3 Group#

KJA# 1940012430 Emergency Procedures/Plan -

Knowledge of which events related to system operations/status that must be reported to internal orginazations or external agencies, such as State, the NRC, or the transmission system operator.

Importance 2.7 4.1 Question # 74 Given:

  • 21 AFW pump failed to automatically start.
  • Manual start of 21 AFW pump was successful.

Which of the following states the NRC Notification Requirements (if any) for this event?

A. No report to NRC Headquarters is required; the NRC Resident Inspector will be informed.

B. NUE will be declared; NRC will be notified viaE-Pian REGS line.

C. Actuation of any unplanned trip when the reactor is critical must be reported to the NRC.

D. The unit trip is not reportable, but the AFW failure to automatically start must be reported to the NRC.

Answer: C

Explanation/Justification:

A. Incorrect. Plausible because the NRC Resident Inspector will be notified, however, actuation of a unit trip is a 4 hour4.62963e-5 days <br />0.00111 hours <br />6.613757e-6 weeks <br />1.522e-6 months <br /> report to the NRC.

B. Incorrect. Plausible because failure of the reactor to trip with manual trip successful is an ALERT. However neither Unit trip nor AFW failure will result in an NUE.

C. Correct. Any event or condition that results in the actuation of the Reactor Protection System when the reactor is critical is a 4 hour4.62963e-5 days <br />0.00111 hours <br />6.613757e-6 weeks <br />1.522e-6 months <br /> report to the NRC.

D. Incorrect. Plausible because any automatic AFW actuation is reportable; however, the reactor trip is reportable Technical

References:

IP-SMM-LI-1 08 Proposed References to be provided: None Learning Objective IOWKB-ILO-ADMOO 2.4.30 Question Source: New Question History: NA Question Cognitive Level: Fundamental Knowledge 10 CRF Part 55 Content: 55.41 (b) 10 Comments

Exam Outline Cross

Reference:

Level RO SRO Tier# 3 Group#

KIA# 1940012225 Equipment Control - Knowledge of bases in technical specifications for limiting conditions for operations and safety limits.

Importance 3.2 4.2 Question# 75 What is the basis for requiring a minimum of 360,000 gallons of water in the Condensate Storage Tank?

Ensures adequate volume to:

A. maintain Hot Zero Power for 24 hours2.777778e-4 days <br />0.00667 hours <br />3.968254e-5 weeks <br />9.132e-6 months <br /> and then cooldown to MODE 4

<350°F.

B. maintain MODE 4 for 24 hours2.777778e-4 days <br />0.00667 hours <br />3.968254e-5 weeks <br />9.132e-6 months <br /> with at minimum of 2 SGs greater than 9%

NR level.

C. Cooldown from Trip concurrent with loss of off-site power to MODE 4

<350°F on RHR.

D. remove decay heat while in MODE 3 for 24 hours2.777778e-4 days <br />0.00667 hours <br />3.968254e-5 weeks <br />9.132e-6 months <br /> following a reactor trip from 102% RTP.

Answer: D

Explanation/Justification:

To satisfy accident analysis assumptions, the CST must contain sufficient cooling water to remove decay heat while in MODE 3 for 24 hours2.777778e-4 days <br />0.00667 hours <br />3.968254e-5 weeks <br />9.132e-6 months <br /> following a reactor trip from 102% RTP.

A. Incorrect. Plausible because the 24 hour2.777778e-4 days <br />0.00667 hours <br />3.968254e-5 weeks <br />9.132e-6 months <br /> time is correct; however, the action and plant conditions are not correct B. Incorrect. Plausible because the 24 hour2.777778e-4 days <br />0.00667 hours <br />3.968254e-5 weeks <br />9.132e-6 months <br /> time is correct; however, the action and plant conditions are not correct C. Incorrect. Plausible because the cooldown with LOOP will be performed using the Atmospheric steam dumps. The CST inventory will be consumed; however, this is not the basis for the MINIMUM volume in the CST.

D. Correct.

Technical

References:

Tech Specs Proposed References to be provided: None Learning Objective 12LP-ILO-CND01 3 13LP-ILO-CND001 9 Question Source: Modified Question History: Unit 3 NRC 2013 Question Cognitive Level: Fundamental Knowledge 10 CRF Part 55 Content: 55.43 (b) 2 Comments

Exam Outline Cross

Reference:

Level RO SRO Tier# 1 Group# 1 KIA# 0000072145 Conduct of Operations -Ability to identify and interpret diverse indications to validate the response of another indication.

Importance 4.3 4.3 Question # 76 The plant experienced a reactor trip from 100% power. The crew completed the immediate operator actions of E-0, Reactor Trip or Safety Injection and transitioned to ES-0.1, Reactor Trip Response.

  • An excessive cooldown resulted in pressurizer level decrease to approximately 12%. Pressurizer Level is now slowly rising.
  • Step 4 of ES-0.1 checks the status of letdown. Both red and green indicating lights for LCV-459 are extinguished.
  • While attempting to replace the green light bulb, the bulb broke and became lodged in the socket.

Which of the following indications can be used to confirm LCV-459 is closed and what actions are directed by ES-0.1? Assume all other equipment operated as designed.

Indications Actions Letdown flow 0 gpm PCV-135 Closed Place letdown in service per 2-SOP-3.1 A.

LP Letdown Relief Temp 102°F when level is greater than 18%.

LP Letdown Pressure 0 psig VCT Level Slowly lowering Place letdown in service per 2-SOP-3.1 B.

Charging Temp Rising slowly when level is greater than 18%.

Letdown flow 0 gpm PCV-135 Closed Manually actuate Sl and go to E-0 step C.

LP Letdown Relief Temp 102°F 1.

LP Letdown Pressure 0 psig Manually actuate Sl and go to E-0 step D. VCT Level slowly lowering 1.

Charging Temp Rising slowly

Answer: A Explanation/Justification:

If all equipment operated as designed, only LCV-459 would have closed.

Indications that letdown flow has stopped confirm that LCV-459 is closed.

Procedurally, ES-0.1 assumes no accident is in progress; therefore, SOP should be used to restore letdown. Also ES-0.1 has a lower threshold for Sl Actuation 9% vice 14% for most other EOPs.

A. Correct B. Incorrect. Plausible because indications for LP Letdown Pressure and VCT level are correct. Charging temp is incorrect. Also the procedure SOP-3.1 is correct.

C. Incorrect. Plausible because all indications are correct. Manually actuating Sl is plausible because most EOP foldout page Sl Actuation Criterion is 14%.

D. Incorrect. Plausible because indications indications for LP Letdown Pressure and VCT level are correct. Charging temp is incorrect. Manually actuating Sl is plausible because most EOP foldout page Sl Actuation Criterion is 14%.

Technical

References:

2-ES-0.1 Proposed References to be provided: None Learning Objective 12LP-ILO-CVCS 5 12LP-ILO-CVCS 8 Question Source: New Question History: NA Question Cognitive Level: Comprehension 10 CRF Part 55 Content: 55.43 (b) 5 Comments

Exam Outline Cross

Reference:

Level RO SRO Tier# 1 Group# 1 KIA# 0000562225 Equipment Control - Knowledge of bases in technical specifications for limiting conditions for operations and safety limits.

Importance 3.2 4.2 Question # 77 The Technical Specifications require that the 6.9 kV bus tie breaker control switches in the CCR be placed in TPO if the 138 kV source is lost and the 13.8 kV source is being used to feed 6.9 kV Buses 5 and 6. What is the basis for this requirement?

A Automatic transfer of 6. 9 kV Buses 1 - 4 to offsite power following a reactor trip could overload the Appendix "R" Diesel if it were carrying the 6.9 kV GT Buses.

B. The 13.8 kV source may not be in sync with the 138kV supply to 6.9 kV Buses 1-4.

C. Automatic transfer of 6.9 kV Buses 1 -4 to offsite power following a reactor trip could overload the 13.8/6.9 kV transformer.

D. Automatic transfer of 6.9 kV Buses 1 - 4 to offsite power following a reactor trip could overload the Gas Turbine if it were carrying the 6.9 kV GT Buses.

Answer: C

Explanation/Justification:

A. Incorrect. Plausible because the Appendix R diesel is a possible source of power to the 6.9kV buses; however, it is not considered part of the 13.8kV offsite circuit.

B. Incorrect. Plausible because it is possible for the 138 kV and 13.8 kV sources to be out of synchronization. A sync-check relay would prevent the tie breakers from closing if this was the concern.

C. Correct.

D. Incorrect. Plausible because a Gas Turbine-Generator is a possible source of power to the 6.9kV buses; however, it is not considered part of the 13.8kV offsite circuit.

Technical

References:

Tech Specs Proposed References to be provided: None Learning Objective 12LP-ILO-EDS01 14 Question Source: Bank Question History: NA Question Cognitive Level: Comprehension 10 CRF Part 55 Content: 55.43 (b) 2 Comments

Exam Outline Cross

Reference:

Level RO SRO Tier# 1 Group# 1 KIA# 1940012418 Emergency Procedures/Plan-Knowledge of the specific bases for EOPs.

Importance 3.3 4 Question # 78 The plant has experienced a SGTR followed by a small break LOCA. Procedure ECA-3.1, "SGTR With Loss Of Reactor Coolant-Subcooled Recovery Desired" has been implemented. While checking if an RCP can be started, the following plant parameters are observed:

  • RCS pressure is 1050 psig.
  • PRZR level is 60% and decreasing.
  • Hot leg temperatures are 500°F and decreasing.
  • Cold leg temperatures are 490°F and decreasing.
  • RCS cooldown rate is 90°F/hr and stable.
  • RCS subcooling based on CETs is 52°F and increasing.
  • RWST level is 19 feet and decreasing rapidly.
  • Containment sump level is 40 feet and increasing.
  • CNMT pressure is 5 psig and increasing.
  • 21 and 22 Sl pumps are running.
  • 23 Sl pump is available.
  • All RCPs are stopped.
  • An NPO reports that a runaway forklift has crashed into the RWST and water is rapidly pouring out of the tank.

Which ONE of the following actions is required for these conditions?

A Transition to ECA-1.2, "LOCA Outside Containment", since this procedure will conserve RWST level.

B. Remain in ECA-3.1 and manually start 23 Sl pump as necessary to restore RCS subcooling.

C. Transition to ECA-3.2, "SGTR With Loss Of Reactor Coolant-Saturated Recovery Desired", since this procedure will minimize RCS leakage.

D. Remain in ECA-3.1 and start an RCP as necessary to restore forced circulation.

Answer: C Explanation/Justification:

Duplicated from question no 25608 with modifications for KA match Explanation:

The appropriate transition is to ECA-3.2 based on the loss of RWST level and sump level below amount needed for recirculation.

SRO justification:

The question requires the candidate to have knowledge of a decision point in ECA-3.1. The candidate has to know there is a need to transition a procedure for this event which is ECA-3.2.

Plausibility:

A. Incorrect but plausible because ECA-1.2 will conserve RWST level and the plant is experiencing a loss of potential recirculation coolant outside containment.

B. Incorrect but plausible it is plausible that Sl flow would be used to maintain subcooling when the cooldown is secured.

C. Correct answer.

D. Incorrect but plausible since forced circulation would be desirable.

Technical

References:

2-ECA-3.1 2-ECA-3.2 Proposed References to be provided: None Learning Objective 12SG-LOR-EOP008 5 Question Source: Bank Question History: None Question Cognitive Level: Comprehension

10 CRF Part 55 Content: 55.43 (b) 5 Comments

Exam Outline Cross

Reference:

Level RO SRO Tier# 1 Group# 1 KIA# 000055A203 Ability to determine and interpret the following as they apply to a Station Blackout: - Actions necessary to restore power Importance 3.9 4.7 Question # 79 Unit 2 is in Mode 3.

A loss of the Station Auxiliary Transformer has just occurred.

All 3 EDGs failed to start.

Con Ed DO stated that 13.8 KV power is available.

Which one of the following is an alignment of 13.8KV to energize the 6.9KV Buses 5 and 6 in AOP-138KV-1?

A From Unit 3 GT35 through Unit 3 GT bus, GTBT and OSP, GT25 and GT26 B. 13W92 through Breakers GT1, GT2, OSP, GT25 and GT26 C. 13W93 through L&P Bus 3, L&P bus 2, and breakers GT1, GT2, OSP, GT25 and GT26 D. 13W94 through Unit 3 GT bus, and breaker GTBT and OSP, GT25 and GT26 Answer: B

Explanation/Justification:

SRO justification:

This is not a systems question since A, C, and D are physically possible; however they are not specifically listed in AOP-138KV-1. The candidate must assess plant conditions and select the correct procedure/section to mitigate the event.

A. Incorrect because this is not specified in the AOP, but plausible since this line up is possible.

B. Correct C. Incorrect because this is not specified in the AOP, but plausible since this line up is possible.

D. Incorrect because this is not specified in the AOP, but plausible since this line up is possible.

Technical

References:

2-AOP-138KV-1 Proposed References to be provided: None Learning Objective 12LP-ILO-EDS01 6 Question Source: Modified Question History: NA Question Cognitive Level: Comprehension 10 CRF Part 55 Content: 55.43 (b) 5 Comments

Exam Outline Cross

Reference:

Level RO SRO Tier# 1 Group# 1 KJA# 000062A202 Ability to determine and interpret the following as they apply to the Loss of Nuclear Service Water: -

The cause of possible SWS loss Importance 2.9 3.6 Question # 80 Given the following:

  • The plant is at 100% power
  • An inadvertent Safety Injection occurs during testing
  • During recovery the BOP notes the following:

o A rise in Containment temperature o Diesel Generator Service Water Flow Low alarm annunciates

  • The Conventional NPO reports the High and High-High Water Temperature Alarms are lit on the EDG Control Panels What actions are required for the above indications?

A. Proper Essential Service Water flow will be re-established when lA is restored or unnecessary SW loads isolated in E-0, Reactor Trip or Safety Injection.

B. Service Water Pumps supplying the essential header will be started in E-O, Reactor Trip or Safety Injection.

C. Fail air to FCV-1176 and FCV-1176A to establish adequate SW flow per 2-SOP-27.3, EDG Manual Operation.

D. Service Water supply valves from the Non-essential header to supply the EDGs only will be opened using 2-SOP-24.1, Service Water System Operation.

Answer: B

Explanation/Justification:

KA Justification:

Per discussion with Dave Silk, knowledge of the cause of the failure is necessary to select the correct action.

A. Incorrect. Loss of instrument air will cause many valves to fail open. In the service water system TCVs for containment fan coolers and FCVs for the EDGs will all fully open on an Sl signal. This will not result in inadequate flow to components.

B. Correct. If the SW Pump Mode Selector Switch is misaligned the pumps supplying the Non-Essential header will start instead of the pumps supplying the Essential Header. Whether using ARP or E-0, service water pumps will be started in the header with no operating pumps. This will restore SW flow to essential components.

C. Incorrect. FCV-1176 and 1176A are SW outlet valves for the EDG Jacket Water Coolers. If these valves are closed, it would cause EDG temperatures to increase; however, this does not explain the rise in containment temperature.

D. Incorrect. A leak could cause inadequate flow; however, for the conditions given securing the EDGs is appropriate.

Technical

References:

2-E-0 2-E-0 BG Proposed References to be provided: None Learning Objective 12LP-ILO-EOPEOO 1 Question Source: Modified Question History: NA Question Cognitive Level: Comprehension 10 CRF Part 55 Content: 55.43 (b) 5 Comments

Exam Outline Cross

Reference:

Level RO SRO Tier# 1 Group# 1 KIA# OOWE05A201 Ability to determine and interpret the following as they apply to the Loss of Secondary Heat Sink: -

Facility conditions and selection of appropriate procedures during abnormal and emergency operations Importance 3.4 4.4 Question# 81 An event caused the plant to trip from 100% power. During the performance of E-O, the operating Team experienced a total loss of main and auxiliary feedwater and has just transitioned to the appropriate functional restoration procedure. The following plant conditions exist:

  • RCS Pressure 800 psig
  • SG Pressures 950 psig
  • SG NR levels Off scale low
  • Steam Driven AFW Pump Out of service for maintenance.
  • Motor Driven AFW pumps Tripped on Over Current Which of the following describe the proper procedure flowpath at this time?

A. Transition to E-1 ,"Loss of Reactor or Secondary Coolant."

B. Attempt to restore flow to the SGs by using AFW, main feed, or condensate systems per the functional restoration procedure.

C. Immediately establish bleed and feed of the RCS per the functional restoration procedure.

D. Transition back to E-0," Reactor Trip or Sl" procedure and step in effect.

Answer: D

Explanation/Justification:

The proper procedure flowpath for this event is E-0 -> FR-H.1 (heat sink not required at step 1) -> E-0 -> E-1.

A. Incorrect. Plausible because E-1 addresses LOCA conditions and will eventually be entered.

B. Incorrect. Plausible because restoring feed flow to the SGs is the first major action category in FR-H.1; however the procedure is exited at step 1 because heat sink is not required.

C. Incorrect. Plausible because bleed and feed criteria is the second major action category in FR-H.1; however the procedure is exited at step 1 because heat sink is not required.

D. Correct. The first step in FR-H.1 evaluates the need for a secondary heat sink.

Heat sink is not necessary for the stated plant conditions.

Technical

References:

2-FR-H.1 3-FR-H.1 Proposed References to be provided: None Learning Objective 12LP-ILO-EOPFH1 1 12LP-ILO-EOPFH1 5 Question Source: Bank Question History: NA Question Cognitive Level: Comprehension 10 CRF Part 55 Content: 55.43 (b) 5 Comments

Exam Outline Cross

Reference:

Level RO SRO Tier# 1 Group# 2 KIA# 0000372237 Equipment Control - Ability to determine operability and/or availability of safety related equipment.

Importance 3.6 4.6 Question # 82 Given the following:

  • The plant was operating at 100% power.
  • NPO reports R-61B, N-16 radiation monitor indicates 150 GPO leak
  • Chemistry confirms the leakrate at approximately 150 GPO
  • During the performance of 2-AOP-RSD-1, Rapid Shutdown, the plant was tripped at 250 MWe.
  • Bus 3A normal supply breaker tripped on overcurrent when the main generator tripped.

Which of the following describes steam generator level control in AOP-SG-1 for the above conditions?

A. 22 AFW pump is used to supply 21 and 22 SG as necessary.

23 AFW pump supplies feedwater to 23 and 24 SGs as necessary.

An NPO is dispatched to close MS-41.

If MS-41 cannot be closed, PCV-1139 is tripped.

B. 21 AFW Pump supplies feedwater to 21 and 22 SG as necessary 23 AFW pump supplies feedwater to 23 and 24 SG as necessary.

22 AFW pump will be used if necessary after MS-41, Steam Supply to 22 AFW pump is closed.

C. PCV-1139, Main Steam Supply to 22 AFW, is tripped.

After MS-41, Steam supply to 22 AFW pump is closed, 22 AFW pump is used to supply 21 SG.

Feed flow to 22 SG is isolated due to leak.

23 AFW pump supplies feedwater to 23 and 24 SGs as necessary.

D. 21 AFW pump supplies feedwater to 21 SG.

Feed flow to 22 SG is isolated due to leak.

23 AFW supplies feedwater to 23 and 24 SG as necessary.

22 AFW pump will be used if necessary after MS-41, Steam Supply to 22 AFW pump is closed.

Answer: A Explanation/Justification:

SRO Only Justification:

Thisquestion is SRO Only because it required knowledge of specific differences between the emergency procedure for SGTR and the abnormal operating procedure for SGTL. In particular, the SGTR procedure will isolate steam to the turbine driven AFW pump until the steam supply from a ruptured SG is isolated as long as one motor driven AFW pump is operating. During a SGTL the turbine driven AFW pump is left running if only one motor driven AFW pump is running and the NPO is dispatched to close the steam supply valve.

A. Correct. 22 AFW pump must be used because bus 3A supplies 21 AFW pump. Feed flow to 22 SG will be isolated ONLY if level is greater than 10% and reinitiate if level decreases to less than 10%.

B. Incorrect. Plausible because 21 AFW pump supplies 21 and 22 SG if power is available. Feed flow to 22 SG will be isolated ONLY if level is greater than 10%

and reinitiate if level decreases to less than 10%.

C. Incorrect. Plausible because 22 AFW pump must be used to feed 21 and 22 because bus 3A supplies 21 AFW pump. Feed flow to 22 SG will be isolated ONLY if level is greater than 10% and reinitiate if level decreases to less than 10%.

D. Incorrect. 21 SG supplies feedwater to 21 SG. Feed flow to 22 SG is isolated due to leak. 23 SG supplies feedwater to 23 and 24 SG as necessary. 22 AFW pump will be used if necessary after MS-41, Steam Supply to 22 AFW pump is closed.

Technical

References:

2-AOP-SG-1 Proposed References to be provided: None Learning Objective 12LP-ILO-AOPSG1 2 12LP-ILO-AOPSG1 3 Question Source: New

Question History: NA Question Cognitive Level: Comprehension 10 CRF Part 55 Content: 55.43 (b) 5 Comments

Exam Outline Cross

Reference:

Level RO SRO Tier# 1 Group# 2 KJA# 1940012106 Conduct of Operations -Ability to manage the control room crew during plant transients.

Importance 3.8 4.8 Question # 83 A fire in the 15ft. control building forced the Team to evacuate the control room and implement AOP-SSD-1, Control Room Inaccessibility Safe Shutdown Control.

Which of the following correctly describes the responsibilities of the team members after the control room has been evacuated?

A. CRS and one RO report to the Nuclear Side.

Second RO and Conventional NPO report to the Aux Boiler Feed Pump Building.

Unit 3 Polisher and Unit 1 NPO conduct electrical lineups.

STA report to TSC to monitor PIGS.

SM provides field oversight.

Unit 3 SM assumes Emergency Director role.

B. CRS and one RO report to the Aux Boiler Feed Pump Building.

Second RO and Conventional NPO report to Nuclear Side.

Unit 3 Polisher and Unit 1 NPO conduct electrical lineups.

STA report to TSC to monitor PICS.

SM assumes Emergency Director role.

Unit 3 SM provides field oversight.

C. CRS and one RO report to the Nuclear Side.

Second RO report to the Aux Boiler Feed Pump Building.

Conventional NPO and Unit 1 NPO conduct electrical lineups.

STA report to TSC to monitor PICS.

SM provides field oversight.

Unit 3 SM assumes Emergency Director role D. CRS and one RO report to the Nuclear Side.

Second RO and Unit 1 NPO report to the Aux Boiler Feed Pump Building.

Unit 3 Polisher and Conventional NPO conduct electrical lineups.

STA provides field oversight.

SM report to TSC to monitor PIGS.

Unit 3 SM assumes Emergency Director role.

Answer: A Explanation/Justification:

The following NOTE appears in 2-AOP-SSD-1 prior to step 4.19.

NOTE The expectations for the use of personnel are as follows:

The CRS, one RO and the Nuclear NPO will be present on the Nuclear Side with one copy of this procedure.

The second RO and the Conventional NPO will be present in the Auxiliary Feed Pump Building. Each person will have one copy of this procedure.

The Unit 3 Polisher Watch and Unit 1 NPO will be conducting electrical operations, etc with the last copy of this procedure (labelled FSS Copy Packet).

STA will report to TSC or Unit 3 CCR to monitor PIGS.

SM provides field oversight.

Unit 3 SM assumes the ED role.

A. Correct B. Incorrect C. Incorrect D. Incorrect.

Technical

References:

2-AOP-SSD-1 Proposed References to be provided: None Learning Objective 12LP-ILO-ASSD 12 Question Source: New Question History: NA Question Cognitive Level: Fundamental Knowledge 10 CRF Part 55 Content: 55.43 (b) 5 Comments

Exam Outline Cross

Reference:

Level RO SRO Tier# 1 Group# 2 KIA# OOWE02A202 Ability to determine and interpret the following as they apply to the Sl Termination:- Adherence to appropriate procedures and operation within the limitations in the facility's license and amendments Importance 3.5 4 Question # 84 Given:

  • The team is performing the actions of ES-1.1, "SI Termination".
  • At step 4 of ES-1.1, The team secured all Sl and RHR Pumps.
  • At step 5 of ES-1.1, the team secured all but one charging pump.
  • RCS Pressure and Pressurizer Level begin to lower after the charging pumps are secured.

Which one of the following describes the required operator actions in accordance with ES-1.1?

A. Monitor RCS Pressure and Pressurizer Level, if pressure and level continue to lower, restart Sl pumps and return to procedure previously in effect.

B. Manually re-initiate Sl and Transition to E-0, "Reactor Trip or Safety Injection" step 1.

C. Adjust charging flow and if Pressurizer Level cannot be maintained, then restart Sl pumps as required and transition to E-1, "Loss of Reactor or Secondary Coolant".

D. Restart one charging pump and if Pressurizer Level cannot be maintained transition to E-1, "Loss of Reactor or Secondary Coolant."

Answer: C Explanation/Justification:

A. Incorrect. Plausible because candidate may believe that returning to the procedure previously in effect will resume mitigation strategies for that event.

B. Incorrect. Plausible because candidate may believe that a new event has occurred and that restarting in E-0 is appropriate.

C. Correct. These are the actions of step 6 in ES-1. 1 D. Incorrect. Plausible because the transition to E-1 is correct; however, the Sl pumps must be started first.

Technical

References:

2-E-1 2-ES-1.1 3-E-1 3-ES-1.1 Proposed References to be provided: None Learning Objective 12LP-ILO-EOPE10 21 Question Source: Bank Question History: IPEC Unit 3 NRC 2013 Question Cognitive Level: Comprehension 10 CRF Part 55 Content: 55.43 (b) 5 Comments

Exam Outline Cross

Reference:

Level RO SRO Tier# 1 Group# 2 KIA# OOWE10A201 Ability to determine and interpret the following as they apply to the Natural Circulation with Steam Void in Vessel with/without RVLIS: - Facility conditions and selection of appropriate procedures during abnormal and emergency operations Importance 3.2 3.9 Question # 85 Given the following plant conditions:

  • The crew was performing ES-0.1, "Reactor Trip Response," when a loss of off-site power occurred.
  • A transition is made to ES-0.2, "Natural Circulation Cooldown," to cool down the plant.

Which ONE of the following identifies (1) a reason the crew would transition to ES-0.3, " Natural Circulation Cooldown with Steam Void In Vessel (with RVLIS)."

AND (2) why RVLIS is maintained greater than 69% during performance of ES-0.3?

A. (1) Unexpected large increase in Pressurizer Level is observed.

(2) To prevent loss of natural circulation due to steam voiding in the SG tubes.

B. (1) Unexpected large increase in Pressurizer Level is observed.

(2) To ensure pressurizer level is adequate to accommodate void collapse.

C. (1) Plant conditions require a cooldown rate faster than allowed by ES-0.2.

(2) To prevent loss of natural circulation due to steam voiding in the SG tubes.

D. (1) Plant conditions require a cooldown rate faster than allowed by ES-0.2.

(2) To ensure pressurizer level is adequate to accommodate void collapse.

Answer: C Explanation/Justification:

A. Incorrect. Plausible because an unexpected large increase in pressurizer level is an indication of a void in the reactor head. Also the reason for maintaining level is correct.

B. Incorrect. Plausible because an unexpected large increase in pressurizer level is an indication of a void in the reactor head. Pressurizer level adequate to accommodate void collapse is a reason for maintaining pressurizer level when starting an RCP.

C. Correct.

D. Incorrect. Plausible because conditions requiring a faster cooldown rate is correct. Pressurizer level adequate for void collapse is not correct, but it is a reason for maintaining pressurizer level when starting an RCP.

Technical

References:

2-ES-0.2 2-ES-0.2 BG 2-ES-0.3 Proposed References to be provided: None Learning Objective 12LP-ILO-EOPS03 1 12LP-ILO-EOPS03 2 Question Source: Bank Question History: Catawba NRC 2010 Question Cognitive Level: Comprehension 10 CRF Part 55 Content: 55.41 (b) 5 Comments

Exam Outline Cross

Reference:

Level RO SRO Tier# 2 Group# 1 KIA# 005000A201 Ability to (a) predict the impacts of the following malfunctions or operations on the RHRS and (b) based on those predictions, use procedures to correct, control, or mitigate the consequences of those malfunctions or operations:

-Failure modes for pressure, flow, pump motor amps, motor temperature, and tank level instrumentation Importance 2.7 2.9 Question # 86 Given the following:

  • A cooldown and depressurization are in progress for refueling.
  • RCS temperature is 210 degrees F and slowly decreasing.
  • RCS pressure is 360 psig and stable.
  • 24 RCP is running.
  • RHR is in service.

The crew observes the following indications:

  • RCS pressure rapidly lowering
  • RHR flow oscillating
  • VCT level increasing Which of the following is a possible cause for the above indications and what procedural actions should be taken?

A. PT-135 Failed High Trip RCP per AOP-RCP-1, Reactor Coolant Pump Malfunction Enter AOP-CVCS-1, CVCS Malfunctions B. PT-135 Failed High Trip RCP if pressure < 325 psig Enter AOP-RHR-1, Loss of RHR

C. PT-402 Failed High Trip RCP per AOP-RCP-1, Reactor Coolant Pump Malfunction Enter AOP-CVCS-1, CVCS Malfunctions D. PT-402 Failed High Trip RCP if pressure < 325 psig Enter AOP-RHR-1, Loss of RHR Answer: B Explanation/Justification:

A. Incorrect. PT-135 failing high will cause PCV 135 to Open. This will cause letdown flow to be greater than charging flow. RCS pressure will decrease rapidly. RCP trip criteria in AOP-RCP-1 may not be reached. AOP-CVCS-1 does address the failure of PCV-135, but it does not address the cavitation of the RHR Pumps.

B. Correct. PT-135 failing high will cause PCV 135 to Open. This will cause letdown flow to be greater than charging flow. RCS pressure will decrease rapidly.

C. Incorrect. PT-402 is interlocked with RHR suction valve 730; however failing high will not cause 730 to close, it will prevent it from opening.

D. Incorrect. PT-402 is interlocked with RHR suction valve 730; however failing high will not cause 730 to close, it will prevent it from opening.

Technical

References:

2-AOP-RHR-1 Proposed References to be provided: None Learning Objective 12LP-ILO-AOPRHR 2 Question Source: New Question History: NA Question Cognitive Level: Comprehension 10 CRF Part 55 Content: 55.43 (b) 5 Comments

Exam Outline Cross

Reference:

Level RO SRO Tier# 2 Group# 1 KIA# 008000A202 Ability to (a) predict the impacts of the following malfunctions or operations on the CCWS and (b) based on those predictions, use procedures to correct, control, or mitigate the consequences of those malfunctions or operations:

- High/low surge tank level Importance 3.2 3.5 Question # 87 Given the following:

  • Unit is operating at 100% reactor power, steady state conditions.
  • All controlling systems are operating normally in automatic.
  • Panel SGF 1-2, Component Cooling Surge Tank Level is in alarm
  • Operators are performing steps in AOP-CCW-1 "Loss of Component Cooling Water" due to a slowly lowering level in the CC surge tank.
  • Makeup is in progress and operators dispatched to locate the leak.

The following sequence of annunciators/indications is received:

  • Panel SGF 1-1 "Component Cooling Pump Discharge Low Pressure".
  • Panel SGF 3-3 "Thermal Barrier CCW Header Low Flow"
  • CCW surge tank is 7% and lowering rapidly The NEXT procedure to be entered by the operators is _ _ (1) _ _ because

_ _ (2) _ _

A. (1) SOP-4.1.1 Component Cooling Filling and Draining (2) normal make-up to the surge tanks is insufficient and backup cooling and makeup are required.

B. (1) ARP-SGF 3-3 (2) overheating of RCP seals will occur due to the loss of thermal barrier cooling if seal injection is subsequently lost.

C. (1) ARP-SGF 1-1 (2) standby CCW pumps will need to be started if they do not auto start to restore header pressure.

D. (1) E-0 Reactor Trip or Safety Injection (2) the reactor must be manually tripped and all RCPs stopped.

Answer: D Explanation/Justification:

Question wording was slightly adjusted based on examiner comments for 2014 exam.

A. Incorrect: Although this procedure does support system makeup cooling to vital loads, it would not address the imminent threat to plant equipment.

Plausible: Some mitigation guidance in contained in these procedures B. Incorrect: Seal failure is not imminent. Charging is providing cooling to seals.

Plausible: ARP could be implemented and does contain guidance to trip reactor and RCP(s) if seal temperature limits are reached.

C. Incorrect: loss of header pressure is a result of the leak so the actions of AOP-CCW should take priority (i.e. trip running CCW pps, trip reactor, trip RCP's)

Plausible: given low header pressure a pump start is a logical response.

D. Correct: Per step 4.12 if surge tank level is approaching off-scale low the per for 4.13-4.16 ... trip CCW pumps, reactor and RCPs.

Technical

References:

2-AOP-CCW-1 Proposed References to be provided: None Learning Objective 12LP-ILO-AOPCCW A 12LP-ILO-AOPCCW C 12LP-ILO-CCW001 11 Question Source: Bank Question History: IPEC Unit 2 NRC 2008 Question Cognitive Level: Comprehension 10 CRF Part 55 Content: 55.43 (b) 5 Comments

Exam Outline Cross

Reference:

Level RO SRO Tier# 2 Group# 1 KIA# 0620002236 Equipment Control - Ability to analyze the effect of maintenance activities, such as degraded power sources, on the status of limiting conditions for operations.

Importance 3.1 4.2 Question # 88 Given the following:

  • Unit 2 is at 100% power.
  • 118 VAC Vital Bus Inverter 22 failed 8 hours9.259259e-5 days <br />0.00222 hours <br />1.322751e-5 weeks <br />3.044e-6 months <br /> ago.
  • Maintenance on 21 EDG is scheduled for this shift Which ONE of the following identifies the minimum Technical Specification LCOs that contain required actions for these plant conditions, including the required compensatory measures in accordance with the Technical Specification Bases?

A. TS 3.8.7, Inverters- Operating, LCO is required to be entered; TS 3.8.9, Distribution Systems- Operating, LCO is NOT required to be entered; Concurrent planned EDG maintenance is NOT allowed.

B. TS 3.8.9, Distribution Systems- Operating, LCO is required to be entered; TS 3.8.7, Inverters- Operating, LCO is NOT required to be entered; Concurrent planned EDG maintenance can be performed.

C. BOTH TS 3.8.9, Distribution Systems- Operating AND TS 3.8.7, Inverters Operating LCOs are required to be entered; Concurrent planned EDG maintenance can be performed.

D. BOTH TS 3.8.9, Distribution Systems- Operating AND TS 3.8.7, Inverters Operating LCOs are required to be entered; Concurrent planned EDG maintenance is NOT allowed.

Answer: A

Explanation/Justification:

A. Correct. The inverter is required to be declared inoperable; however, the bus is still energized and operable. The basis is correct; limiting maintenance activities with an inverter unavailable reduces the likely hood of an inadvertent ESF actuation (due to momentary power interruption) in the event of a LOOP during the time the inverter is inoperable.

B. Incorrect. Plausible because LCO entry is reversed and relationship between EDGs and Instrument Buses is not obvious.

C. Incorrect. Plausible because the LCO for inverters is required, but the LCO for Distribution Systems is not. The relationship between EDGs and Instrument Buses is not obvious.

D. Incorrect. Plausible because the LCO for inverters is required, but the LCO for Distribution Systems is not. The relationship between EDGs and Instrument Buses is not obvious.

Technical

References:

Tech Specs Proposed References to be provided: None Learning Objective 12LP-ILO-EDS04 6 Question Source: Bank Question History: NA Question Cognitive Level: Comprehension 10 CRF Part 55 Content: 55.43 (b) 2 Comments

Exam Outline Cross

Reference:

Level RO SRO Tier# 2 Group# 1 KIA# 1940012408 Emergency Procedures/Plan -

Knowledge of how abnormal operating procedures are used in conjunction with EOPs.

Importance 3.8 4.5 Question # 89 Given:

  • A seismic event has occurred.
  • A small break LOCA has occurred inside the VC.
  • Damage to the intake structure has resulted in a catastrophic failure of the Service Water System.
  • VC pressure is 9 psig and rising.
  • The crew is performing actions in E-1, Loss of Reactor or Secondary Coolant.
  • Off-Site Power is available Which of the following describes the actions for containment cooling?

A. Perform FR-Z. 1, Response to High Containment Pressure, in parallel with E-1 to re-establish containment cooling using Containment Spray System.

B. Perform 2-AOP-SW-1, Service Water Malfunction, in parallel with E-1 to establish Service Water from the Wash Water system to the Fan Cooler Units.

C. Perform 2-SOP-22.1, Wash Water System and Traveling Water Screen actions in parallel with E-1 to augment Service Water with Wash Water.

D. Perform 2-SOP-ESP-1, Local Equipment Operation and Contingency Actions in parallel with E-1 to supply Service Water from the Fire Water System.

Answer: B

Explanation/Justification:

A. Incorrect. Plausible because FR-Z.1 will address challenges that result from inadequate containment cooling; however, rule of usage does not allow performing a functional restoration procedure in parallel with an EOP.

B. Correct. Rules of usage does allow performing AOPs in parallel with EOP if they do not interfere with the actions in the EOP and if their actions are necessary.

C. Incorrect. Plausible because SOP-22.1 is the procedure for normal lineup and operation of the Wash Water System. SOP-22.1 does not contain directions for augmenting the Service Water system with Wash Water.

D. Incorrect. Plausible because SOP-ESP-1 contains directions for various contingency actions and plant system lineups. Fire water can be aligned for a variety of uses if it is necessary; however, it can not be aligned to augment service water.

Technical

References:

2-AOP-SW-1 Proposed References to be provided: None Learning Objective 12LP-ILO-AOPSW1 2 Question Source: New Question History: NA Question Cognitive Level: Comprehension 10 CRF Part 55 Content: 55.43 (b) 5 Comments

Exam Outline Cross

Reference:

Level RO SRO Tier# 2 Group# 1 KIA# 103000A203 Ability to (a) predict the impacts of the following malfunctions or operations on the Containment System and (b) based on those predictions, use procedures to correct, control, or mitigate the consequences of those malfunctions or operations: -

Phase A and B isolation Importance 3.5 3.8 Question # 90 The following conditions exist at Unit 2:

  • A large break loss of coolant event occurred
  • Containment pressure peaked at 26 psig and is currently 3 psig and stable
  • No HHSI pumps are available
  • All 6.9KV and 480V busses were continuously powered from offsite power
  • The team has just exited E-0 and FR-C.1 entry criteria are met
  • The average of the 5 highest CETs is greater than 1200°F
  • All procedural actions have been completed for the above conditions Which of the following describes how the RCPs will be operated assuming no other equipment changes?

A. If SG level is less than 10% narrow range, RCPs will be started without regard to support conditions.

B. 24 RCP will be secured to conserve for possible future use.

C. If SG level is greater than 10% narrow range, RCPs will be started without regard to support conditions.

D. RCPs will remain operating without regard to plant conditions.

Answer: C

Explanation/Justification:

Candidate must first determine the condition of the RCPs -- Secured due to phase B. Then the candidate must assess conditions and select the appropriate section of FR-C.1 to mitigate the event.

A. Incorrect. Plausible because all RCPs are expected to be secured and RCPs are started without regard to RCP support conditions; however SG level must be greater than 10%.

B. Incorrect. Plausible because RCP trip criteria requires a safety injection pump operating prior to tripping the pumps assuming CCW is still available to the pumps. Since a phase B has occurred, the pumps should be tripped. Also FR-C.2 will secure only 24 RCP to save it.

C. Correct. RCPs would have been tripped due to the Phase Band regardless of safety injection pump status. Pump support conditions are not necessary to start the RCPs, however SG level must be > 10% narrow range.

D. Incorrect. Plausible because RCP trip criteria requires a safety injection pump operating prior to tripping the pumps assuming CCW is still available to the pumps. Since a phase B has occurred, the pumps should be tripped.

Technical

References:

2-E-0 2-FR-C.1 Proposed References to be provided: None Learning Objective 12LP-ILO-EOPFC1 5 Question Source: New Question History: None Question Cognitive Level: Comprehension 10 CRF Part 55 Content: 55.41 (b) 5 Comments

Exam Outline Cross

Reference:

Level RO SRO Tier# 2 Group# 2 KIA# 034000A201 Ability to (a) predict the impacts of the following malfunctions or operations on the Fuel Handling System and (b) based on those predictions, use procedures to correct, control, or mitigate the consequences of those malfunctions or operations: -

Dropped fuel element Importance 3.6 4.4 Question# 91 During core offload, the first fuel assembly was being removed from the core.

The gripper was not properly engaged in the top nozzle so when the assembly was raised approximately halfway out of core it dropped back in.

  • The gripper is in the latched position with no assembly attached.
  • No permanent or temporary radiation monitor readings changed.
  • Fuel damage cannot be determined since the assembly is surrounded on all four sides.
  • There are no gas bubbles coming from core.

Which of the following describes the required actions (if any) per AOP-FH-1, Fuel Damage or Loss of SFP/Refueling Cavity Level, and what actions are needed to retrieve the fuel assembly?

A. Containment shall be evacuated.

The gripper can be unlatched in its current position to allow for retrieval of the fuel assembly.

B. Containment evacuation is not required.

The gripper can be unlatched in its current position to allow for retrieval of the fuel assembly.

C. Containment shall be evacuated.

The gripper must be placed down on assembly or unlatch fixture to allow unlatching prior to retrieval of the fuel assembly.

D. Containment evacuation is not required.

The gripper must be placed down on assembly or unlatch fixture to allow unlatching prior to retrieval of the fuel assembly.

Answer: 0 Explanation/Justification:

Plausibility/explanation:

AOP-FH-1 section for fuel damage in the VC only specifies evacuation if radiation monitors alarm. However, it is very plausible that a candidate could think that we would be required to evacuate the VC on a dropped assembly.

This makes A and C incorrect.

The gripper will only unlatch if it is sitting on something that pushes a mechanical interlocking device up. It is plausible that candidate does not know this. Also, there is an electrical interlock prevent gripper unlatching if there is weight detected on the hoist. This could make a candidate believe that unlatching is possible since there is no weight.

This makes A and B incorrect.

Technical

References:

2-AOP-FH-1 Proposed References to be provided: None Learning Objective 12LP-ILO-FHD001 11 Question Source: New Question History: NA Question Cognitive Level: Comprehension 10 CRF Part 55 Content: 55.41 (b) 7 Comments

Exam Outline Cross

Reference:

Level RO SRO Tier# 2 Group# 2 KIA# 0720002418 Emergency Procedures/Plan -

Knowledge of the specific bases for EOPs.

Importance 3.3 4 Question # 92 Given:

  • The plant experienced a SBLOCA.
  • Containment pressure spiked at 8 psig
  • Containment pressure is currently 2 psig.
  • R-25 and R-26 both spiked at 3 X 105 R/hr
  • R-25 and R-26 currently indicate 4 X 103 R/hr Which of the following describes the process parameter values (Normal vs Adverse Containment Values) used in the EOPs?

A. Use Normal Values; both process parameters are currently below the value for Adverse Containment. Post accident instrument deviations are relatively insignificant below 3 psig and 105 R/hr.

B Use Normal Values: only containment pressure exceeded the setpoint and it is currently below the value for Adverse Containment. Post accident instrument deviations are relatively insignificant below 3 psig and 10 5 R/hr.

C. Use Adverse Values; containment radiation exceeded and remains above the setpoint. Post accident instrument deviations due to radiation are dependent on current existing radiation level.

D. Use Adverse Values; both process parameters exceeded the setpoints and are currently below the value for Adverse Containment. Post accident instrument deviations due to radiation are based on accumulated dose.

Answer: D

Explanation/Justification:

SRO Only Justification:

This question is SRO only because it requires WOG EOP Background Executive Volume knowledge of when adverse containment values are used and when their use can be terminated. In particular for containment pressure, adverse values are used as long as condition exists and return to normal values when the condition no longer exists. If the Adverse containment values for high radiation are exceeded, the CRS never returns to normal values until an engineering evaluation is performed.

A. Incorrect. Plausible because if only containment pressure exceeds its setpoint and returns below, the CRS should return to using Normal Values.

B. Incorrect. Plausible because if only containment pressure exceeds its setpoint and returns below, the CRS should return to using Normal Values.

C. Incorrect. Plausible because Adverse Values should be used any time radiation exceeds it value; however, post accident deviations are the result of cumulated radiation dose to instruments.

D.Correct.

Technical

References:

WOG-Executive Proposed References to be provided: None Learning Objective 12LP-ILO-EOPROU 23 Question Source: New Question History: NA Question Cognitive Level: Comprehension 10 CRF Part 55 Content: 55.41 (b) 5 Comments

Exam Outline Cross

Reference:

Level RO SRO Tier# 2 Group# 2 KIA# 035000A201 Ability to (a) predict the impacts of the following malfunctions or operations on the S/GS and (b) based on those predictions, use procedures to correct, control, or mitigate the consequences of those malfunctions or operations:

-Faulted or ruptured S/Gs Importance 4.5 4.6 Question # 93 Given the following conditions:

  • The unit was operating at 100% power
  • Nuclear Plant Operators are currently trying to close any MSIV
  • The Control Room team has entered ECA-2.1, "Uncontrolled Depressurization of all Steam Generators" and just completed step 2 to throttle AFW flow.
  • An NPO reports that the 22 Steam Generator MSIV has been closed Which of the following describes the expected plant response and actions to be taken for the above conditions?

A. 22 SG pressure will increase and the crew should remain in ECA-2.1 until Sl is terminated.

B. 22 SG pressure will increase and the crew should transition to E-2, "Faulted Steam Generator Isolation".

C. 22 SG pressure will continue to decrease due to cooldown and the crew should remain in ECA-2.1 until Sl is terminated.

D. 22 SG pressure will continue to decrease due to cooldown and the crew should transition to E-2, "Faulted Steam Generator Isolation".

Answer: 8

Explanation/Justification:

A. Incorrect. Plausible because pressure will increase and the foldout page states that if Sl termination is in progress, it should continue.

B. Correct. SG pressure will increase and an SRO should know that step 2 is too early to begin Sl termination. The foldout page instructs a transition to E-2 if pressure begins to increase in any SG unless Sl termination actions are in progress.

C. Incorrect. Plausible because a large cooldown of RCS could cause an isolated SG pressure to increase; however, since AFW is throttled, this will not occur. In addition, the foldout page states that if Sl termination is in progress, it should continue.

D. Incorrect. Plausible because a large cooldown of RCS could cause an isolated SG pressure to increase; however, since AFW is throttled, this will not occur.

In addition, the transition to E-2 is correct.

Technical

References:

2-ECA-2.1 Proposed References to be provided: None Learning Objective 12LP-ILO-EOPC21 2 12LP-ILO-EOPC21 1 Question Source: Modified Question History: NA Question Cognitive Level: Comprehension 10 CRF Part 55 Content: 55.43 (b) 5 Comments

Exam Outline Cross

Reference:

Level RO SRO Tier# 3 Group#

KIA# 1940012125 Conduct of Operations - Ability to interpret reference materials such as graphs, curves, tables etc.

Importance 3.9 4.2 Question # 94 Given:

  • The reactor vessel head is being removed to replace a leaking 0 ring.
  • RCS level is at 66' 1" elevation.
  • Both trains of RHR are initially in service.
  • 95' Equipment Hatch is removed and in storage location.
  • RCS temperature is 108°F and stable.
  • RHR Suction Valve 730 goes closed and operators secure both RHR Pumps.

Using Graph ACS-2C, determine time to reach 200°F due to the loss of RHR.

Additionally determine what actions (if any) AOP-RHR-1 would require regarding the 95' Hatch.

A. 25 minutes. Containment closure is not required because the RCS is not in Reduced Inventory.

B. 25 minutes. The Equipment Hatch or Closure Plug must be in place to close containment before this time elapses.

C. 30 minutes. Containment closure is not required because the RCS is not in Reduced Inventory.

D. 30 minutes. The Equipment Hatch or Closure Plug must be in place to close containment before this time elapses.

Answer: B

Explanation/Justification:

KA match, SRO justification. The question meets the KA because the candidate has to use the graph to determine heatup rate. The question is an SRO question because the candidate has to assess condition and then determine the appropriate section of the AOP-RHR-1 that applies.

A. Incorrect but plausible. Based on the graph, it will take 25.2 min to reach 2000F. The RCS is also above reduced inventory (<66'), however, AOP-RHR-1 requires containment closure within 30 minutes or time to 2000F, whichever is shorter.

B. Correct answer.

C. Incorrect but plausible. A miscalculation or misread of graph could lead to this value. See "A" for further explanation of this choice.

D. Incorrect but plausible. The only wrong information is the time.

Technical

References:

2-AOP-RHR-1 Proposed References to be provided: None Learning Objective 12LP-ILO-AOPRHR 3 Question Source: New Question History: NA Question Cognitive Level: Comprehension 10 CRF Part 55 Content: 55.41 (b) 5 Comments

Exam Outline Cross

Reference:

Level RO SRO Tier# 3 Group#

KIA# 1940012136 Conduct of Operations-Knowledge of procedures and limitations involved in core alterations.

Importance 3 4.1 Question # 95 Unit 2 is in Mode 6 with core alterations in progress.

As per 2-SOP-17.30, Manipulator Crane Operations, which ONE of the following, by title, must approve bypass of a Fuel Handling interlock NOT specified in accordance with procedures for routine fuel handling activities?

A Shift Manager B. Refueling SRO C. Fuel Handling Supervisor D. Shift Outage Manager Answer: B

Explanation/Justification:

Question originates from McGuire 2008 NRC Exam SRO Question Unit 1 is in Mode 6, core alterations are in progress.

Which ONE of the following, by title, must approve bypass of a Fuel Handling interlock not specified in accordance with procedures for routine fuel handling activities?

A. Shift Manager B. Fuel Handling SRO C. Refueling Supervisor D. Reactor Engineer Proposed Answer: B Explanation:

A. Incorrect. Shift Manager responsible for unit, but FH SRO is responsible for all refueling activities B. Correct.

C. Incorrect. Administrative oversight required, but not approval for FH bypass D. Incorrect. Shift Outage Managers will be involved in the critical path decisions, but are not part of approval for FH bypass; they are only approval authority during physics testing Technical

References:

2-SOP-17.30 Proposed References to be provided: None Learning Objective 12LP-ILO-FHD001 20 Question Source: Bank Question History: McGuire 2008 Question Cognitive Level: Fundamental Knowledge

10 CRF Part 55 Content: 55.43 (b) 7 Comments

Exam Outline Cross

Reference:

Level RO SRO Tier# 3 Group#

KIA# 1940012217 Equipment Control - Knowledge of the process for managing maintenance activities during power operations, such as risk assessments, work prioritazation, and coordination with the transmission system operator.

Importance 2.6 3.8 Question # 96 Given:

  • Unit 2 is at 100% Power with no equipment out of service.
  • Unit 3 is in a refueling outage
  • 138kV feeder 33332 L&M is out of service
  • The quarterly tests, 2-PT-0013 lnservice Valve Tests are scheduled for today for 1176 and 1176A.

Can each of the valve stroke tests be performed? Why or why not?

A. No. Stroke testing 1176 and 1176A will cause the EDGs to be sequentially inoperable. On Line Risk Assessment will prevent performance of this test.

B. Yes. Stroke testing 1176 and 1176A will cause the EDGs to be sequentially inoperable. On Line Risk Assessment will allow performance of this test.

C. No. Stroke testing 1176 and 1176A will NOT cause the EDGs to be inoperable. On Line Risk Assessment will prevent performance of this test.

D. Yes. Stroke testing 1176 and 1176A will NOT cause the EDGs to be inoperable. On Line Risk Assessment will allow performance of this test.

Answer: D

Explanation/Justification:

A. Incorrect. Plausible because the existing plant conditions remove TS required 13.8kV power to both Unit 2 and 3. It may appear that any testing related to the EDGs would not be allowed; however EN-WM-104 definition of AVAILABLE would allow the test. Stroke testing of 1176 and 1176A will not cause EDGs to be inoperable.

B. Incorrect. Plausible because the existing plant conditions remove 138kV power between Unit 2 and 3. It may appear that any testing related to the EDGs would not be allowed; however EN-WM-104 definition of AVAILABLE would allow the test. Stroke testing of 1176 and 1176A will not cause EDGs to be inoperable.

C. Incorrect. Plausible because the existing plant conditions remove 138kV power between Unit 2 and 3. It may appear that any testing related to the EDGs would not be allowed; however EN-WM-104 definition of AVAILABLE would allow the test. Stroke testing of 1176 and 1176A will not cause EDGs to be inoperable.

D. Correct. Testing 1176 and 1176A do not cause the EDGs to be inoperable.

Therefore there is minimal increase in risk related to the test.

Technical

References:

EN-WM-104 Proposed References to be provided: None Learning Objective IOLP-ILO-ADM01 2.2.17 Question Source: New Question History: NA Question Cognitive Level: Comprehension 10 CRF Part 55 Content: 55.43 (b) 2 Comments

Exam Outline Cross

Reference:

Level RO SRO Tier# 3 Group#

KIA# 1940012218 Equipment Control - Knowledge of the process for managing maintenance activities during shutdown operations, such as risk assessments, work prioritization, etc.

Importance 2.6 3.9 Question # 97 The following conditions exist at the end of a forced outage:

  • RCS Temperature is 325°F.
  • All equipment required by tech specs for this mode is operable.
  • Both trains of RHR cooling are in service for core cooling.
  • Shutdown Risk is Green for all Safety Functions.
  • The plant will be on hold in this condition for 48 hours5.555556e-4 days <br />0.0133 hours <br />7.936508e-5 weeks <br />1.8264e-5 months <br /> awaiting repairs to a Main Steam Isolation Valve Actuator.
  • Because of the delay, the Outage Control Center has decided to scope in a repair to piping from the CST causing AFW to be unavailable.

Which of the following is correct per IP-SMM-OU-1 04, Shutdown Risk Assessment, for this added work activity and the next 48 hours5.555556e-4 days <br />0.0133 hours <br />7.936508e-5 weeks <br />1.8264e-5 months <br />?

A. Risk will have to be formally assessed due to AFW becoming unavailable.

Risk will NOT have to be routinely assessed for the next 48 hours5.555556e-4 days <br />0.0133 hours <br />7.936508e-5 weeks <br />1.8264e-5 months <br /> unless there is an equipment status change.

B. Risk will have to be formally assessed due to AFW becoming unavailable.

Risk will be assessed shiftly while the plant remains in the current mode.

C. Because tech specs do not require AFW in the current mode, risk does NOT have to formally assessed when making it unavailable. Risk will be assessed shiftly while the plant remains in the current mode.

D. Shutdown Risk is Green and will remain Green since SGs are not being used for core cooling. Risk is only required to routinely assessed when Shutdown Risk is Yellow or above.

Answer: B Explanation/Justification:

Explanation:

As per IP-SMM-OU-104, the SM/designee is responsible for completing the Shiftly Outage Risk Assessment whenever the plant is in Modes 4,5,6 and defueled. Risk is also reassessed when risk significant equipment is taken out of service. AFW is not required by TS for this mode, whether or not SGs are available for heat sink is part of the risk calculation. Since SGs are not available without AFW, this action will cause the Core Cooling function to go from Green to Yellow. Because of this change, the shiftly risk form will be filled out since the existing one is no longer current.

SRO question justification:

The KA is clearly met and answering the question requires a solid understanding of how risk is managed during outages. Shutdown risk assessment is an "SRO Only" task at Indian Point.

Plausibility:

A. The first statement is correct. It is plausible that the form would only need to be completed when conditions change.

B. Correct answer.

C. Since TS does not require AFW, it is plausible that no actions would be required. The second part is correct.

D. Since AFW and SGs are not currently required forTS, is reasonable to assume that it would not affect risk. It is plausible that the form is only filled out for Yellow or above conditions since there are actions that are only taken for Yellow or above conditions (e.g. defense in depth contingency plans)

Technical

References:

IP-SMM-OU-1 04 Proposed References to be provided: None Learning Objective IOLP-ILO-ADM01 3 Question Source: New Question History: NA

Question Cognitive Level: Fundamental Knowledge 10 CRF Part 55 Content: 55.43 (b) 5 Comments

Exam Outline Cross

Reference:

Level RO SRO Tier# 3 Group#

KIA# 1940012304 Radiological Controls-Knowledge of radiation exposure limits under normal and emergency conditions.

Importance 3.2 3.7 Question# 98 Current Conditions:

  • A Site Area Emergency has been declared due to a LOCA outside Containment.
  • The LOCA is into the PAB building and a pathway to the environment exists.
  • Limited makeup to the RWST is available.
  • An operator has volunteered to go to the PAB building to locally isolate the leak.
  • The EOF has been fully staffed and operational for an hour.
  • This action will result in a significant reduction in offsite dose, protecting a large population.

What is the maximum emergency exposure this operator may receive while performing this action?

A. 10 Rem TEDE with SM approval B. 25 Rem TEDE with SM approval C. 10 Rem TEDE with ED approval D. 25 Rem TEDE with ED approval Answer: D

Explanation/Justification:

Modified from question no 18694 with changes to be similar to a Seabrook NRC exam question EP-Form 6 is the reference A. Incorrect but plausible since this is the emergency limit for protecting valuable property. It is plausible that the SM could authorize this exposure since he is ED qualified.

B. Incorrect but plausible. The dose is correct and is plausible that the SM could authorize this exposure since he is ED qualified.

C. Incorrect but plausible since this is the emergency limit for protecting valuable property.

D. Correct answer per IPEC E-Pian Form 6 Technical

References:

IP-EP-115 Proposed References to be provided: None Learning Objective IOLP-ILO-ERT005 14 Question Source: Bank Question History: Seabrook 2009 Question Cognitive Level: Comprehension 10 CRF Part 55 Content: 55.43 (b) 4 Comments

Exam Outline Cross

Reference:

Level RO SRO Tier# 3 Group#

KIA# 1940012432 Emergency Procedures/Plan -

Knowledge of operator response to loss of all annunciators.

Importance 3.6 4 Question # 99 The following conditions exist at Unit 2:

A Station Auxiliary Transformer fault occurred.

Bus 6A is de-energized and the cause is being investigated.

A trip occurred because 21 RDMG set was secured prior to the event.

DC circuit trips have caused a loss of all supervisory panel annunciators.

All instrument busses are energized.

All other equipment operated as designed.

Which of the following describes the proper emergency plan classification for this event?

A. NUE due to the loss of offsite AC power sources.

B. ALERT due to the loss of offsite AC power sources and 6A not being energized.

C. NUE due to the loss of annunciators.

D. ALERT due to the loss of annunciators with a significant transient in progress.

Answer: D

Explanation/Justification:

The question tests SRO level knowledge of selecting the appropriate procedure section of the Emergency Plan for the event based on given conditions. While it is not expected that a candidate have the EAL chart memorized, recognizing this classification from memory is reasonable. Statements are in the stem saying that alternate monitoring is available to prevent SAE from being a valid answer. This has the added benefit of making the NUE for annunciators and electrical EAL answers more plausible.

A. Incorrect but plausible because without the loss of annunciators, this would be the correct call.

B. Incorrect but plausible because this is the correct call if the plant was limited to one available power supply.

C. Incorrect but plausible since a candidate may not interpret a trip as a significant transient in progress. The plant would be considered stable for these conditions. However, for the purposes of this EAL call, a trip is considered a significant transient.

D. Correct answer.

Technical

References:

IP-EP-120 Proposed References to be provided: None Learning Objective IOLP-ILO-ERT002 8 Question Source: New Question History: None Question Cognitive Level: Comprehension 10 CRF Part 55 Content: 55.41 (b) 5 Comments

Exam Outline Cross

Reference:

Level RO SRO Tier# 3 Group#

KIA# 1940012446 Emergency Procedures/Plan -

Ability to verify that the alarms are consistent with the plant conditions.

Importance 4.2 4.2 Question# 100 The following conditions exist at Unit 2:

  • A loss of coolant accident occurred.
  • All equipment was available and operated as designed.
  • Transfer to cold leg recirculation was completed 90 minutes after the initiating event.
  • The team is at the last step of ES-1.4 and completed transfer to hot leg recirculation within the appropriate time period.
  • 21 Recirculation Pump, 21 and 23 HHSI Pumps are in service.
  • The ATC just pointed out that the "LOW HEAD INJECTION LOW FLOW 320 GPM" alarm is up.

What actions should the CRS take at this time?

A. The CRS should transition back to E-1, Loss of Reactor or Secondary Coolant.

B. The CRS should transition back to ES-1.3, Transfer to Cold Leg Recirculation.

C. The CRS should direct throttling open HCV-638 and/or 640 to increase low head flow.

D. The CRS should transition to ECA-1.3, Loss of Emergency Coolant Recirculation Caused by Sump Blockage.

Answer: A

Explanation/Justification:

Alarm is up because ES-1.4 will close Valves 746 and 747 which isolates low head flow. This will stop low head flow. All flow will be via the HHSI pumps.

When transfer to cold leg recirculation was completed, ES-1.3 would have directed the crew back to E-1. When transfer to hot leg recirculation is complete, ES-1.4 directs the crew back to procedure and step in effect. The procedure would be E-1.

A. Correct answer B. Incorrect because transition is to E-1. Plausible because a candidate may not know that E-1 is re-entered after transfer to cold leg recirculation. In fact, B would be the correct answer for IP-3.

C. Incorrect because the alarm is expected. Plausible because a candidate may not know that this alarm is expected and the actions are reasonable.

D. Incorrect because alarm is expected. Plausible because a candidate may not know that this alarm is expected and the actions are reasonable.

Technical

References:

2-ES-1.4 Proposed References to be provided: None Learning Objective 12LP-ILO-EOPS14 1 Question Source: New Question History: NA Question Cognitive Level: Comprehension 10 CRF Part 55 Content: 55.43 (b) 5 Comments